sexta-feira, 27 de março de 2015

acerto

Professor, então o que você acha sobre o livro do Dr. Joseph Murphy O Poder do Subconsciente, que aborta a magia do nosso cérebro em construir nosso futuro.‎

A mais completa baboseira. Não existe força do pensamento positivo. O que algum pensamento positivo pode é criar disposições para agir. Mas é a ação que resolve. Força advém de ações e não de intenções.

"Sem um sujeito vivente não pode existir qualquer tempo ou qualquer espaço." O que acha?‎

Completamente falso. O espaço e o tempo existem independentemente de qualquer vida. Se não tivesse surgido vida no Universo eles existiriam da mesma forma. A única diferença é que não seriam percebidos por mente alguma. Note que a percepção do espaço do tempo não requer apenas vida, mas vida dotada de sistema nervoso capaz de ter percepção do mundo exterior. Estou falando de percepção e não de sensação.

tentar justificar a não existência de Deus por meio de argumentos cientificos não se trata de cientifismo? já que a ciência só pode explicar o que existe no mundo e Deus é extrínseco a ele, caso exista.‎

Note que a ciência não justifica nem explica a inexistência de Deus. O que faz é considerar que não existe uma vez que não há nada que justifique ou explique que exista. Supor que exista é, pois, uma consideração gratuita. Por outro lado, por definição de mundo (ou melhor, Universo), não é possível existir nada que não pertença a ele, pois ele é o conjunto de tudo o que existe. Se existir, Deus faz parte do Universo. Portanto a consideração de sua existência é da competência da ciência sim.

Professor, você acredita no poder da mente subconsciente ?‎

Se você se refere a um poder capaz de influenciar diretamente o mundo exterior, isso não existe. O que existe é o poder do inconsciente de influir sobre a consciência de modo a que ela tome decisões, quando as toma, sugeridas pelo inconsciente. Todavia, na maior parte dos casos, é o inconsciente que toma as decisões e as comunica à consciência (se não em todos os casos). Essa é a influência. Mas a influência sobre o mundo exterior só acontece quando a mente ativa o organismo através dos nervos eferentes para que a musculatura e os órgãos executem movimentos (entre eles o falar). Por meio da ação muscular é que nosso organismo interfere no mundo exterior. O pensamento apenas, não convertido em ações, não causa nenhuma ação fora da própria mente que o pensa.

Professor, você acredita em sonhos? Quando sonhamos com algo, e realiza ou é pura coincidência ?‎

Sonhos existem e refletem o que a mente considera relevante para a vida da pessoa. Mas não são premonições. Não há nada que relacione o que se sonha com o que vai acontecer. Pode ser que aconteça porque o cérebro é capaz de intuir possibilidades mais prováveis.

Bom dia, Ernesto. Como está? Vi que está procurado me contactar.‎

Olá Luiza. Tudo bem comigo. Sempre estou te procurando, porque você some ou muda de identidade. E você é uma referência para mim. Um porto seguro. Uma âncora. Suas palavras são um refrigério, um bálsamo. Me embriago de sua sabedoria. Fico contente em ver que está bem e que está amando. Nada é mais significativo do que o amor. Beijos, querida amiga.

O que há além do universo observável?‎

O que há além do universo observável?‎  Leinadv Se o Universo for infinito ou se for finito e maior do que o Universo Observável, além desse existe, simplesmente, mais universo. Note que o Universo Observável é um subconjunto do Universo que varia com o ponto e o momento da observação. Quem estiver em outro lugar do Universo, diferente do nosso, observará porções que nós não observamos e vice-versa. Da mesma forma que nós, no passado, observávamos uma porção menor do Universo e, no futuro, observaremos uma porção maior. O tamanho do Universo observável está crescendo por dois motivos. Porque, com a passagem do tempo, mais e mais porções do Universo atingem o ponto de onde a luz teve tempo de chegar até nós e porque o próprio espaço está se expandindo. Por isso é que, apesar da idade do Universo ser de 13,8 bilhões da anos, o raio do Universo Observável não é 13,8 bilhões de anos-luz, mas 92 bilhões de anos-luz. É que, desde que a luz saiu de lá e vem caminhando para cá, o Universo foi se expandindo. O cálculo pode ser encontrado em:
http://dournac.org/info/size_universe

Como e por que o universo se expande?‎

Supõe-se que a expansão tenha sido provocada por algum campo, denominado "inflaton", que atuou no próprio espaço, inchando-o. Não atuando mais esse campo, ela se arrefece, devido à gravidade. Tal campo foi muito atuante na época conhecida como "inflação", que se deu entre 10^-36 a 10^-32 segundos depois do Big Bang e aumentou o fator de escala do Universo em 10^26 vezes. De onde veio esse campo e porque ele apareceu não se sabe. Outra questão a respeito da expansão é que ela está acelerando, ao invés de desacelerando, como seria previsto pela ação da gravidade. Tal aceleração seria causada por outro campo, denominado (impropriamente) "energia escura". Veja isto:
http://en.wikipedia.org/wiki/Inflation_(cosmology)
http://ned.ipac.caltech.edu/level5/Watson/Watson_contents.html
http://en.wikipedia.org/wiki/Dark_energy

Se o universo for finito, como uma esfera como você disse abaixo, o que teria depois da esfera? De fora, sempre vai ter que haver algo depois

Veja bem. Falei da esfera como uma analogia bidimensional do universo tridimensional. Se você for um ser bidimensional, sobre a superfície da esfera, não existe nada além dessa superfície. Não existe o espaço de fora e nem de dentro da superfície. Ao longo da superfície, você não encontra fronteira nenhuma. Então não existe "depois" da esfera. Considerando isso no caso tridimensional seria o mesmo, só que em três dimensões. Não existiria, caso o universo fosse finito, uma fronteira dele, além da qual se estaria "fora" do universo. Não existe "lado de fora" do universo. Partindo de qualquer ponto e indo sempre para frente, nunca se achará uma barreira. Vai-se avançando até ver que se chega de onde se saiu, vindo por trás. Não existe espaço vazio dentro do qual o universo se situa. Todo o espaço existente está no universo. E não existe lugar vazio no universo. Todo lugar é preenchido por campo e radiação. Matéria pode haver ou não. Quando não há tem-se o vácuo. Mas nunca se tem lugar vazio. Campo e radiação sempre estão presentes, haja ou não matéria.

Na sua opinião, qual é o melhor argumento contra os direitos dos animais? E qual o melhor contra os direitos das crianças?‎

Você é mesmo irônico. Não existe argumento contra isso, e você sabe. Realmente não aprecio nem um pouco a ironia.

Oque o senhor achou da ideia apresentado sobre nossa evolução no filme interestelar?

Ainda não assisti esse filme. Estou esperando ser disponibilizado, já que não assisto filme pirata e ele ainda não foi exibido em Viçosa.

Não entendi como poderia ser que o Universo todo fosse menor do que o Universo Observável e ser possível ver além do tamanho do Universo.‎

Se o Universo for finito, ele tem que ser fechado. Como a superfície de uma esfera, em duas dimensões, é fechada. Ou seja, ela não se estende indefinidamente, mas, indo-se sempre para frente sobre ela, volta-se a onde se partiu, vindo por trás. Só que isso tem que ser entendido em três dimensões e não só duas. Nessa analogia bidimensional, contudo, o Universo Observável seria a parte da superfície esférica, em relação a um ponto dela, da qual se poderia mandar um sinal que fosse recebido nesse ponto, indo com a velocidade da luz. Tal região seria uma calota centrada no ponto, como se ele fosse um pólo, indo desse ponto até um círculo tal que o comprimento do meridiano que passe pelo pólo e vá até esse círculo, como um paralelo da Terra, meça a distância que a luz percorreria desde que esse universo tenha passado a existir. Além disso a luz não teria tido tempo de chegar até o ponto, pois teria que percorrer uma distância maior ainda. Dependendo do tamanho desse universo bidimensional e de seu tempo de vida, esse círculo iria crescendo a partir do ponto, à medida que o tempo iria se passando, atingiria um valor máximo (o equador) e, então iria diminuindo de novo até chegar ao pólo oposto. Depois disso o ultrapassaria e começaria a crescer de novo, o arco de meridiano passando de 180°. Então o universo observável, começaria a ser maior do que o próprio universo. Isso poderia acontecer sem limite, a distância ao longo do meridiano podendo dar várias voltas em torno da esfera. Claro que, enquanto isso fosse acontecendo, o próprio tamanho da esfera iria crescendo, pois o universo que ela representa está em expansão. Tudo vai depender se esse crescimento se daria com velocidade maior ou menor do que a da luz. Isso não é proibido para a expansão cósmica, pois ela não se refere a nada com conteúdo de massa e energia, mas ao próprio espaço.

Professor, afinal, temos ou não livre-arbítrio? Se somos matéria e junção de átomos, e átomos não pensam, não somos apenas fruto das leis naturais da física? Não havendo assim, liberdade?‎

Átomos não pensam, mas estruturas de átomos pensam. A complexidade produz novas situações que a simplicidade não contempla. Todavia, por mais complexos que sejamos, se a natureza fosse determinista não teríamos livre arbítrio. E o temos. Isto é uma constatação fática. Há a possibilidade de escolha e não só para humanos. Outros animais também a apresentam em variáveis graus. Até uma formiga. Mas não vegetais. Porque não possuem sistema nervoso. O indeterminismo intrínseco da natureza é que permite que a estrutura e o funcionamento do sistema nervoso possibilite escolhas e a existência da liberdade.

Estudar matemática ajuda a ser bom em ciências humanas?‎

Sim, se se estudar matemática mesmo e não ficar treinando resolver problemas do tipo que caem nos exames. Porque matemática é, principalmente, lógica e lógica é essencial para qualquer atividade intelectual. Saber demonstrar fatos matemáticos é saber fazer uso da lógica. E isso é preciso para se argumentar qualquer assunto, seja matemático, físico, químico, biológico, filosófico, sociológico, político, econômico, administrativo, linguístico, musical, poético, pictórico, o que for. Especialmente um bom desempenho matemático facilita muito a dialética e a retórica que, além da gramática e da semântica, são essenciais para o traquejo linguístico. Note: o importante não é saber macetes de solução de problemas matemáticos. É saber fazer demonstrações, especialmente inéditas. O que os professores precisam é treinar os alunos a "provar" e não a "resolver" problemas.

O que acha da resposta do Guilherme e o que tem a dizer sobre a pergunta que eu fiz a ele? http://ask.fm/GuiTomishiyo/answer/125597001044‎

Discordo do Tomishiyo, A Mecânica Quântica não diz que as possibilidades físicas do estado de um sistema existem em universos separados. A "Interpretação de Muitos Mundos" de Everett e DeWitt é que propõe que assim o seja. Mas ela não é confirmada (nem rejeitada). Todavia considero muito implausível. O que considero, como a maioria dos físicos, é que as possibilidades são virtuais e quando se faz uma observação, uma delas é revelada, as outras ficando perdidas. Enquanto nenhuma observação seja feita, nenhuma delas pertence à realidade. Em lugar nenhum. Veja isto:
http://pt.wikipedia.org/wiki/Interpreta%C3%A7%C3%A3o_de_muitos_mundos
http://arxiv.org/abs/gr-qc/9703089
http://www.mth.kcl.ac.uk/~streater/lostcauses.html#XII

Professor, objetos estritamente bidimensionais só existem virtualmente, como fotos em um computador, não é? Tudo que existe materialmente tem pelo menos "um átomo" de altura, largura e comprimento. Estou pensando certo?‎

Sim. É isso mesmo. Na natureza tudo que seja substancial, isto é, feito de algo, é tridimensional. Todavia podem haver entidades naturais não substanciais que sejam bidimensionais, como a interface entre dois meios em um sistema de fases separadas, como a interface entre um sólido e um gás em que esteja imerso. A interface é a superfície de separação, que não pertence nem a um nem a outro meio. Ela não tem espessura e não é feita de matéria nem campos. Mas ela existe. Nem tudo que existe é substancial, isto é, feito de matéria, radiação ou campos. Isso não quer dizer que seja só uma abstração ou que seja sobrenatural (que, se existir, não seria abstrato). É real, natural e não substancial. E não é um fato, um evento, uma ocorrência. É uma "coisa". Uma parte muito interessante da metafísica é a categorização da realidade. O que é uma "coisa"? O que é um "objeto"? O que é um "fato"? O que é um "evento"? O que é uma "relação"? O que é um "ente"? O que é um "ser"? O que é um "valor"? O que é uma "lei"? E assim por diante.

Sempre gostou do seu nome, ou aprendeu a gostar dele?‎

Desde que sei como me chamo, gosto do meu nome. Depois que soube porque assim fui chamado, isto é, porque é o nome dos meus dois avôs e o que ele significa, isto é, "sério", gosto mais ainda. Sou fã de meus avôs. O pai da minha mãe era oficial médico da Marinha do Brasil e trabalhava na pesquisa da cura e na erradicação da doença beribéri, da qual morreu, na região do pantanal e do rio Paraná. Morreu como almirante. Era primo primeiro de Rui Barbosa. O pai do meu pai era professor de Russo e Tcheco na Universidade de Viena, além de saber mais seis idiomas: alemão, inglês, francês, italiano, espanhol e português (de Portugal). Apesar de ser da nobreza austríaca, era republicano e a favor da independência da Hungria, da Eslováquia e da Nação Tcheca do Império Austríaco. Tinha idéias anarquistas e veio para o Brasil como imigrante, em 1906, para não se tornar prisioneiro político, quem sabe, até, condenado à morte. O pai dele foi professor de química do Arquiduque Francisco Ferdinando, cujo assassinato deu começo à primeira guerra mundial. Meu pseudônimo Wolfedler é uma homenagem a meu avô paterno, que se chamava Wolfgang Anton Ferdinand Ernst Ginzel Edler von Rückert.

O multiverso é uma teoria plausível? De que seriam constituídos outros universos?‎

Não acho que seja não. E nem há nenhum indício de que exista. Trata-se de uma conjectura hipotética advinda da extensão das soluções das equações cosmológicas e da física de partículas, normalmente rejeitadas por não corresponderem a situações confirmadas. Pode ser que exista, mas, por enquanto, é só conjectura. Contudo, não acho que exista. Isso é o que acontece, também, com as supercordas e as branas.

Oi sr. Ernesto, tudo bom? Vc gosta de futebol? Que estilo de futebol te agrada mais, do Messi ou Cristiano Ronaldo?

Não gosto de futebol. Nunca gostei, nunca joguei, nunca assisti, não entendo nada a respeito. Mas não acho que não se possa gostar. Não aprecio esporte nenhum, especialmente se for jogado com bola e em times. Ainda aprecio um pouco de alpinismo, trilha a cavalo, patinação no gelo, ginástica olímpica e alguns outros do tipo. O que eu gosto menos nos esportes é o fato de haver competição. Se fosse só a atividade física sem disputa, eu apreciaria alguns. Sem bola.

Se o universo sempre foi infinito, como ele no início tinha um raio menor do que um tamanho de um elétron? A não ser que esteja se referindo ao conteúdo do próprio universo.‎

O que tinha o raio menor do que um elétron, ao começar a expansão, não era o Universo e sim o Universo Observável, que hoje tem um raio de 92 bilhões da anos-luz. O Universo todo, se é infinito, sempre foi infinito. A expansão cósmica não é um aumento do tamanho do Universo e sim da separação entre seus lugares. Se o Universo fosse finito, o aumento da separação entre os lugares significaria, também, um aumento do tamanho do Universo. Inclusive, nesse caso, poderia ser que o Universo todo fosse menor do que o Universo Observável. Ou seja, seria possível ver além do tamanho do Universo, isto é, olhar para frente e ver a nossa nuca através da luz que desse a volta pelo Universo. E isso poderia ser feito mais de uma vez.

Morrer é gostoso?‎

Não conheço a opinião de nenhum morto a respeito.

Qual sua opinião sobre traição em um relacionamento? É errado namorar mas ficar com outros, sabendo que isso não altera o seu sentimento pelo seu companheiro?‎

Traição é um erro não porque seja uma falta de exclusividade relacional, mas porque se trata de uma enganação. Ou seja, quem trai, trai porque mente e não porque se relaciona com mais de uma pessoa. Se essas relações múltiplas forem do conhecimento e da aprovação de todos os envolvidos não há traição. Se duas pessoas namoram e consideram que elas podem ficar livremente com quem quiserem, ou mesmo terem mais de um namorado ou uma namorada simultâneos, não há traição quando isso ocorrer. E isso pode acontecer envolvendo sentimento de amor sincero e profundo pelas pessoas todas. Note que o que se está falando é sobre relacionamentos e não sobre amor. Porque amor não há como evitar que aconteça de modo nenhum. E ele pode acontecer por mais de uma outra pessoa sim, sendo muito mais comum do que se pensa que seja. A questão é que se costuma considerar que, acontecendo, é preciso se optar pelo relacionamento com apenas um deles, desistindo do outro. Isso é que é causa de muita infelicidade e não há razão nenhuma para que não possa haver, exceto porque se acha que não se possa. Trata-se de uma moral preconceituosa que precisa ser revogada, como já o foram várias outras, por exemplo a exigência de virgindade até o casamento para moças.

Tu não acha que se o universo fosse perfeito, sem maldade, sem catástrofes, doenças e afins, o mundo não teria uma super população?‎

Não necessariamente, pois as espécies poderiam ter desenvolvido, pela evolução, algum mecanismo de controle de natalidade, de modo que, à medida que a população crescesse, a fertilidade diminuiria. Isso faria parte da perfeição do Universo.

Podemos concluir que há uma criação sem dispor de dados adicionais sobre o criador? Ex: um quadro sem a assinatura do pintor, ainda assim é uma criação, uma manifestação. O fato de não existir tal assinatura significa que o pintor nunca existiu? Que o quadro foi pintado pelo "acaso"?

Um quadro é um artefato, isto é, um objeto produzido artificialmente. Isso requer um autor. A natureza não requer um autor. Todas as suas estruturas podem surgir por acaso e, de fato, assim o foram. Isso pode ser observado atualmente à medida que se formam, por exemplo, o contorno das costas litorâneas, as florestas, os desertos e uma série de outras estruturas. Bem como o surgimento das espécies vivas, umas a partir das outras. Todos esses eventos permitem inferir que o restante também foi surgindo por injunções aleatórias. Isso se aplica aos planetas, às estrelas, ás galáxias e, também, ao Universo como um todo.

Ernesto, vc é cantor e compositor, sabe tocar qual instrumento musical?

Estudei três anos de piano e consigo tocar um pouco. Mas, desde que me casei, não tenho mais piano em casa, de modo que estou destreinado.

Existe algum problema de cunho filosófico no que se trata o fisicalismo? Seria ela a escola/doutrina/posicionamento filosófico "mais correto"?

Fisicalismo é a concepção de que a realidade exterior à mente, isto é, tudo o que exista por si mesmo e não como um construto abstrato, uma ideia, seja exclusivamente física, ou seja, composta pelos elementos físicos do Universo, isto é, campo, matéria, radiação, espaço, tempo e suas estruturas e ocorrências. Isso exclui da realidade objetiva os espíritos e assemelhados, como deuses e almas. A questão de ser o fisicalismo a verdade ou não não é uma questão filosófica e sim uma questão fenomenológica e, pois, científica. Trata-se, portanto, de se proceder uma verificação fática. Se se ficar evidente ou demonstrado que não existe nenhuma realidade sobrenatural, o fisicalismo será verdade. Caso contrário, é falso. Até o momento não se conseguiu demonstrar a existência de nenhuma categoria de realidades sobrenatural. Isso não demonstra, por outro lado, que elas não existam. Mas é uma forte indicação de que não existam. Por outro lado, uma vez que não se verifica que existam, a hipótese básica é a suposição de sua não existência. Daí o Fisicalismo dever ser suposto como a concepção correta a respeito da natureza da realidade objetiva do mundo. Ou seja, de que ela seja totalmente física.

Ernesto, quando buscamos no mundo natural - e principalmente em estruturas biológicas - sinais de planejamento, funcionalidade e propósito, isso significa que, dizer que o homem pode existir sem Deus é dizer que um relógio pode existir(ou ser construido) sem o relojoeiro?

Não. Porque um relógio é um artefato, isto é, um objeto construído e não natural. Objetos naturais podem surgir sem um construtor. Podem surgir como resultado de uma causa ou não. Como não são planejados, não há propósito e nem intenção em seu surgimento, podendo ser qualquer coisa. No caso de objetos complexos, como um ser humano, o surgimento é resultante de um processo evolutivo gradual ao longo de centenas de milhões de anos, em que cada etapa foi fortuita, mas ficou estabelecida pela seleção natural, que não é fortuita.

Mas e o coração no infarto, fica acelerado?‎

Não necessariamente. Mas pode acontecer, especialmente se a pessoa ficar muito impressionada com o fato de estar tendo um infarto, como geralmente fica, já que pode ser fatal.

Quem você gostaria que fosse o presidente do Brasil ?

Cristóvam Buarque

Do que é feito o campo?

Campo é a entidade substancial básica da natureza. Ou seja, ele não é feito de nada mais primitivo do que ele. Ele é aquilo de que tudo o mais é feito. Radiação e matéria são quantizações bosônicas e fermiônicas de campo. Todas as propriedades dessas quantizações e de seus conglomerados advém das propriedades dos campos de que são construídos, como energia, momento linear, momento angular, carga elétrica e tudo o mais.

bom dia Professor. bem, sou de uma época em que se amava apenas uma vez e que se eram felizes para sempre. então Professor, o que foi que mudou? o amor agora é uma mercadoria que se vende ou se troca com prazo de validade e tudo?‎

Isso acontecia e acontece ainda. Há pessoas que amam uma única outra pela vida toda. Tanto antes quanto agora. Mas há muitas outras que, agora e antes, amam a mais de uma, quer sequencialmente, quer em paralelo. Só que não se admitia isso de uma forma aberta e franca, com a aceitação desses amores configurados em relacionamentos, quer pela alternância quer pela pluralidade. Isso provocava infelicidade, pois as pessoas ou precisavam continuar a viver uma relação em que não mais havia amor ou teriam que optar por uma relação só, quando havia mais de um amor. Amor não se controla. Ele surge espontaneamente. Impedir que todos os amores sejam realizados em relações consentidas é uma violência. Amores múltiplos sempre existiram. O que mudou foi que, atualmente, isso está passando a ser aceito como normal, sem nenhum problema. E isso é que é bom, para a felicidade das pessoas. Tal situação não tem nada a ver com dizer que amor é uma mercadoria que se vende, se compra, se aluga ou se troca. Amor, de verdade, não é nada disso. Amor só se dá e se recebe de graça. O que se pode comprar, vender, trocar ou alugar é a situação do relacionamento. Isso tem que acabar. Para isso é preciso, em primeiro lugar, que nenhuma pessoa humana adulta dependa economicamente de outra. Depois, que se acabe a formalização dos relacionamentos, que passariam a ser inteiramente livres. Então as pessoas só os manteriam por gosto. Assim é que tem que ser. Tirar completamente o aspecto econômico dos relacionamentos gâmicos. Isso, em absoluto, não enfraquece a família e nem prejudica as relações de paternidade e maternidade. Basta que se considere a família como tem que ser considerada, isto é, um grupo de pessoas ligadas por laços de parentesco sanguíneo e, principalmente, de afeto, que se apoiam mutuamente, que se comprazem em compartilhar a vida em comum, que colaboram entre si para o sustento de todos. Podem morar todos juntos ou não. Podem ter as ligações amorosas e sexuais que quiserem. Nada sendo estabelecido por contrato. Isso é que é uma verdadeira família.

Se alguem perguntasse ao senhor qual maior argumento que usaria pra dizer a um cristão que não há nenhum deus acima de nós?‎

Não só para um cristão mas também para qualquer um que ache que Deus exista, seja de que religião for, ou mesmo que não tenha religião, há vários argumentos contra essa existência ou, pelo menos, contra a existência de um Deus com as características que a maior parte das religiões atribui a seu conceito. Para começar, a ser Deus, há que ser onipotente, senão não seria Deus. Então o argumento de Epicuro mostra, cabalmente, que, existindo Deus, ele não pode ser bondoso. Veja isto:
http://pt.wikipedia.org/wiki/Paradoxo_de_Epicuro
Além disso, há outras considerações.
A imperfeição do Universo (existem doenças, por exemplo), depõe contra a consideração de que seria necessário um Deus para explicar a perfeição do Universo.
A não necessidade de causa para todo evento, mostrada pela Física Quântica, depõe contra a consideração que seria necessário um Deus para ser a causa primeira da existência de tudo.
O fato de que existência não seja um atributo do ser, mas uma situação, derruba o argumento ontológico para a existência de Deus. Veja isto:
http://pt.wikipedia.org/wiki/Argumento_ontol%C3%B3gico
Fé, sendo uma crença em assertivas não evidentes, não comprovadas e sem plausibilidade, não pode se sustentar como critério de verdade de nada. Portanto basear a veracidade da existência de Deus na Fé é completamente despropositado.
Há quem considere que a existência de Deus seja um requisito para haver qualquer critério ético de comportamento. Isso não procede, pois a ética se fundamenta em considerações puramente humanistas.
O fato de que a ciência tem obtido, ao longo da história, cada vez mais explicações naturais para muitos fatos considerados como de origem sobrenatural, como o surgimento da vida e do Universo, leva a considerar que, mesmo o que ainda não se obteve explicação, um dia se obterá, não se podendo recorrer a Deus para o preenchimento de lacunas de explicação. O que não se sabe, simplesmente "ainda" não se sabe.
Há outras razões, que no momento não me ocorrem. Além dessas razões para não se acreditar em Deus, há, também, razões para não se acreditar na existência de uma alma espiritual e nem de que tal alma seja imortal.

Como a inteligência se manifesta pela primeira vez no estado físico?

Respondi na outra pergunta. Todavia quero comentar que, ao perguntar sobre a inteligência "no estado físico", você, implicitamente, considera que ela possa haver fora do estado físico. Acontece que não existe inteligência em nenhum sistema que não seja físico. Por enquanto, inclusive, em nenhum que não seja biológico, especificamente animal. Sistemas não físicos seriam sistemas abstratos ou sobrenaturais. Os primeiros não apresentam inteligência pois, eles mesmos, são construtos mentais de sistemas físicos inteligentes. Quanto aos segundos, não existem.

O que eu realmente gostaria que me fosse descrito com consistência, é como a inteligência se manifesta pela primeira vez no mundo físico, quais os seus antecedentes, quais os fatores que propiciaram uma primeira e milagrosa manifestação de discernimento intelectual, de consciência de existir. No me

Inteligência é um atributo mental. Mente é a ocorrência de um cérebro (e seus anexos) em funcionamento. A inteligência é o atributo da mente que lhe capacita a solucionar problemas e a aprender. Só animais possuem inteligência. Ela se desenvolveu gradativamente ao longo da evolução. Não há uma espécie específica a partir da qual algum ramo evolutivo se tornou inteligente. Todos os ramos evolutivos dos animais apresentam inteligência, que pode ir aumentando à medida que a evolução acontece (ou não). Inteligência não é nada milagroso. É uma ocorrência normal. Consciência também. Consciência é o atributo mental do ser saber que é algo distinto do resto do mundo, bem como de saber que está pensando e o que está pensando. Pensamento é um processamento mental interno, isto é, não concernente às sensações nem aos comandos motores e glandulares. Recentemente discorri sobre isso aqui no Ask. Consciência, como inteligência, surgiu gradativamente ao longo da evolução. Também não é milagre nenhum. Por enquanto não existe inteligência e nem consciência exceto em sistemas biológicos animais. Mas pode ser que, no futuro, isso se apresente em artefatos.

Ernesto, por que nenhum brasileiro ganhou o premio Nobel da Paz?

Porque não apareceu, ainda, um que merecesse.

Quando se tem dificuldade em matemática e física, é uma má escolha querer cursar Física - bacharelado?‎

A princípio sim, pois a Física requer muita matemática para ser desenvolvida. E não é só essa matemática elementar do nível médio. Todavia é possível se superar essa dificuldade se a pessoa tiver muito interesse e muita dedicação ao estudo.

Sobre Física: Por que quando a luz é refratada, e passa a ter uma velocidade menor, o raio se aproxima da reta normal?‎

Ele se aproxima, justamente, porque a velocidade líquida da luz em algum meio é menor. Se fosse maior, ele se afastaria. Isso pode ser verificado pela aplicação do "Princípio de Huygens" à progressão das frentes de onda ao se passar de um meio para outro.

Professor, vc se considera uma pessoa "acida" pelo fato de abalar a fé de quem acreditar em Deus e também em extraterrestres?

O que se entende por "ácida"? Não me acho ácido, pois não intento corroer o bem estar de ninguém. Pelo contrário, o que intento é livrar as pessoas de crenças infundadas, para que possam encarar o mundo a partir de uma cosmovisão estabelecida com base em evidências e comprovações e não em suposições e crenças. Isso é que é libertador e fomentador de uma maior consciência do significado da vida e da existência de tudo. LIbertar-se das crenças religiosas e outras, como nas visitas de extraterrestres, sem que os fatos sejam corroborados, é a atitude mais saudável que se possa ter. Ao promover essa aquisição de uma saúde mental, livre desse tipo de praga, não estou sendo ácido e sim doce.

Jesus era judeu?‎

Sim.

Esse campo (escalar?) pré big bang de que tanto falas é o campo de Higgs ou Inflation?

O campo primordial do Universo que passou a se expandir no Big Bang era um campo indiferenciado, completamente genérico, do qual, por quebras sucessivas de simetria, à medida que a expansão se dava, passou a se diferenciar nos diversos campos e a se quantizar nas diversas partículas. Tal campo poderia já existir imperturbável antes que o tempo iniciasse sua existência ou ter surgido imediatamente antes do início da expansão, quando o tempo começou a passar. Por enquanto não há como se verificar o que se deu. Nem tampouco se tem aparato algum que produza a densidade de energia capaz de provocar seu surgimento artificial. O LHC chega apenas à densidade suficiente para estabelecer o campo de Higgs, o que já é muito, mas não o que existia no começo do Big Bang.

O que o senhor acha? Nosso pulmão tem tamanho e peso. O oxigênio que respiramos também é mensurável. Tem seu tamanho e peso. Nosso cérebro tem tamanho e peso, mas o pensamento, nem tamanho, nem peso. Ou se pode pesar o pensamento. Não se pode medir a bondade, nem engarrafa-la. Não se pode dobra-la‎

Claro que pensamentos e sentimentos não possuem volume e massa, pois não são substanciais, nem materiais nem constituídos de radiação ou campos. Pensamentos e sentimentos são ocorrências que se dão em certas estruturas. São acontecimentos. Memórias também não são substanciais, são estruturais. Isto é, são feitas de conexões neuronais. Portanto também não possuem volume nem massa. Todavia o pensamento, as emoções e os sentimentos, bem como as decisões e volições podem ser detectados em imageria cerebral pelo consumo de energia que dispendem, o quem incrementa o fluxo sanguíneo na região cerebral em que estejam ocorrendo. Todavia as técnicas de imageria ainda não conseguem detectar seus conteúdos.

Por que esse maluca que disse que o sol é frio é levado a deboche e Olavo de Carvalho a sério por um monte de gente (leia-se direitistas). Seriam todos eles lunáticos?‎

Olavo de Carvalho é outro desses. Só que tem uma aura de seriedade. Mas é tão maluco quanto o Yezzi. Apenas um maluco erudito.

"Porque as pessoas boas escolhem as pessoas erradas?"‎

Isso não é verdade. Pessoas boas ou más podem escolher pessoas certas ou erradas. É uma questão de acertos ou desacertos, coincidências favoráveis ou desfavoráveis, decisões corretas ou incorretas. Não há nada de definitivo nisso.

Eu li num lugar que fótons obrigatoriamente devem possuir massa, caso contrário violariam a equação de Einstein, possuindo energia nula. Isto não procede certo?‎

Claro que não. Não é preciso possuir massa para ter energia. A equação E = mc² relaciona o quanto de energia pode ser obtida pela redução de massa em uma reação nuclear ou o quanto de massa um sistema passa a ter em função de suas energias internas. Mas não tem nada a ver com o fato de poder haver energia sem massa, como é o caso da radiação eletromagnética.

porque falam que o universo está esfriando sendo que tambem falam que ele está se expandindo aceleradamente?‎

Os dois fatos acontecem e o esfriamento decorre, justamente, da expansão. Como o Universo é um sistema adiabático, sua energia é constante e um aumento do volume implica em uma diminuição da densidade de energia que é, justamente, a temperatura.

Mas aí cada área específica perde em profundidade. Horrível.‎

Não perde nada. Por que perderia? Uma questão de geografia, por exemplo, pode envolver um complexo cálculo matemático. Ou uma questão de matemática, conhecimentos de física ou química. Geralmente a questão interdisciplinar possui uma disciplina mestra e outras acessórias. Nessa disciplina mestra ela pode ser tão profunda quanto se queira. Um problema de física pode, por exemplo, envolver um trabalho de interpretação de texto bem complexo, que exija conhecimento de algum tipo de vocabulário de alguma outra área.

E o sofrimento que precede a morte, não te amedronta? Além da possibilidade de ficar em coma muito tempo antes da morte também.‎

O sofrimento deve ser ruim, mas, na maior parte dos casos ele é bem breve. Quanto ao estado de coma, se for inconsciente, não tem nenhum problema para quem está sofrendo.

Em uma perspectiva científica, na psicologia, o que seria a "reflexão" e o "pensamento"? De que forma ambos acontecem e por que fazemos tais?‎

Pensamento é todo tipo de atividade psíquica interna, isto é, não relacionada aos sentidos nem às atividades motoras e glandulares. Envolve conceituação, análise, juízo, reflexão, raciocínio, emoção, sentimento, escolha, volição, decisão e toda atividade feita pela mente apenas dentro de si mesma, bem como os processos de gravação e evocação de memórias. Tais atividades se dão por troca de impulsos entre neurônios que não se requeiram interação com os nervos aferentes nem eferentes. A maior parte do pensamento é inconsciente e constitui a intuição. O pensamento consciente é aquele que se sabe que está ocorrendo. Não são apenas os seres humanos que pensam. Com maior ou menor grau de complexidade, os animais também pensam.

Questões interdisciplinares do tipo misturar química, física e biologia na mesma questão? Isso é um absurdo.‎

Absurdo nenhum. Assim é que tem que ser. Não só física, química e biologia mas também geografia, história, português, inglês e tudo o mais. O conhecimento tem que ser aferido em bloco. Por exemplo, uma questão de física que exija matemática e geografia e que seja redigida em inglês. Assim é que era o ENEM quando começou. Assim fazendo o ENEM pressionará as escolas a se adaptarem à forma interdisciplinar de ensino, com muito maior benefício para os estudantes. E forçará os professores a se reciclarem nas outras matérias além da sua.

Um feixe de luz emitido a partir de uma pulsar distante, que rotaciona rapidamente, é mais veloz que a velocidade da luz?‎

Não. De modo nenhum. Ele é emitido exatamente com a velocidade da luz, independentemente do pulsar estar ou não em rotação e com qualquer velocidade angular. O que pode se mover com velocidade maior do que a da luz são os pontos atingidos pelo feixe no espaço, à medida que ele se desloca para o lado. Mas isso não envolve nenhuma questão relativística, pois esses pontos não são sistemas físicos que possuam massa ou energia. Se se mandar um feixe de laser para a Lua e se girar rapidamente o dispositivo emissor, o ponto iluminado lá na Lua pode se deslocar para o lado com velocidade maior do que a da luz, sem contrariar a relatividade, pois não haverá nada que tenha massa ou energia se movendo com essa velocidade. Só a região geométrica iluminada é que estará se deslocando. O próprio solo da Lua e a luz que o atinge não estarão com velocidade maior do que a da luz.

O que acha da abordagem do ENEM?Não digo do nível das questões, mas da forma como são feitas, exigindo bastante da competência leitora.‎

Isso é ótimo. Tem que ser assim mesmo. Mas acho que deveria ser como no começo do ENEM, isto é, questões completamente interdisciplinares sem pontuação separada por área. Todavia há algumas questões que a resposta pode ser obtida sem se ler o texto. Nesse caso o texto deveria ser omitido. Sou totalmente contra questões só de memorização ou que se resolvam apenas por fórmulas. O importante é aferir a capacidade de interpretação, de raciocínio, de síntese. Cada questão tem que exigir, conhecimento, entendimento, compreensão e habilidade de aplicação. Assim fazendo o ENEM sinalizará para o Ensino Médio e Fundamental que o treinamento para resolver questões dos antigos vestibulares não vale mais. As questões têm que ser elaboradas por experts que evitem a todo custo que quem domine "macetes", "marretas" e similares não logre acertá-las. Só quem, de fato, conheça, entenda, compreenda e tenha habilidade em aplicar o conteúdo em situações inusitadas seja capaz de acertá-las. Assim o ENEM estará promovendo uma eficaz reforma da Educação Básica em poucos anos. Uma reforma que passa a tratar a educação como educação e não como instrução.

Professor, li que no início da Filosofia, acreditar nos mitos precisava ter fé, já que não era racional. Então podemos dizer que a existência de Deus é um mito?‎

Certamente que sim.

http://ask.fm/wolfedler/answer/125241135901 Professor, mas na verdade, é justamente quem acredita em Deus que não se importa em morrer porque acredita na imortalidade da alma. Os ateus por outro lado, não acreditam. Portanto, deveriam sentir muito medo, já que sua existencia acabaria para sempre.

Mas qual o problema de deixar de existir? Não tenho medo nenhum de morrer. Simplesmente deixarei de existir.

Diabetes tem cura?‎

Não, mas pode ser controlado:
http://pt.wikipedia.org/wiki/Diabetes_mellitus

Oque acha do Che Guevara? Qual eh a sua opiniao sobre ele?‎

Não admiro nenhum revolucionário, mesmo que pretenda o que quer que seja de melhor para o mundo. Acho que a mudança do mundo para um lugar bom tem que ser feita por um processo evolutivo e não por nenhuma revolução.

O que acontece, na sua opinião, depois que uma pessoa morre? Tudo simplesmente se apaga para o pessoa? Ou acha que a consciência vai além?‎

Já respondi várias vezes. Ela deixa de existir. Apaga tudo. A consciência desaparece. Como num sono profundo, num desmaio ou numa anestesia. Só que em definitivo. A pessoa não existe mais, logo também não existe sua mente nem sua consciência. O que sobra são seus átomos, que se integrarão à natureza de alguma forma. Não há nenhuma evidência de que não seja isso o que aconteça. A consciência é uma função da mente, que é uma ocorrência do organismo, que não pode existir sem ele funcionando.

Ernesto, faz muito tempo que você não posta vídeo no seu canal do youtube. Na minha opinião um tópico interessante seria ''Dicas de estudo''. O que você acha?‎

Realmente quero postar mais vídeos. Mas não gosto muito dessas "dicas de estudo". Sempre achei que o estudo tem que ser uma atividade muito bem sedimentada e feito com grande dedicação, sem a preocupação de se valer de nenhum estratagema para aprender com menor esforço. É, justamente, o esforço desprendido que confere uma significância e uma valorização ao estudo e faz com que ele seja proveitoso em termos de um aprendizado sedimentado e consistente. A pressa é contrária ao bom resultado. A eficácia fica perdida se se pretender muita eficiência.

Professor, o termo "buraco de minhoca"(wormhole em inglês) foi criado pelo físico teórico estadunidense John Archibald Wheeler em 1957. Existe realmente essa possibilidade de viajar distancias enormes no continuo espaço tempo ou não passa apenas de uma característica topológica hipotética?

Os "buracos de minhoca" são hipotéticos. Não há evidência nenhuma de sua existência real.

Professor, a humanidade já descobriu tudo o que poderia ser descoberto?

Claro que não. Ainda falta descobrir muito mais do que se sabe. Esse "muito" são vários milhares de vezes. Se ainda existirmos por uns 15 milhões de anos, durante esse tempo todo estaremos sempre descobrindo mais e mais. E numa progressão geométrica. As possibilidades do saber são incomensuráveis, de tão grandes. Depois de 200 mil anos de existência, ainda estamos na primeira infância de nossa evolução cultural como espécie.

"Se você quiser saber da alma humana, não deve perguntar a nenhum psiquiatra, psicólogo ou psicanalista. Deve perguntar aos poetas, aos escritores. São eles os que mais sabem, os que mais podem dizer sobre a mente humana." (Freud) - O que pensa? Por que?‎

Concordo. Os escritores e poetas renomados assim o são, justamente, porque possuem uma fina capacidade de perceber as nuances do psiquismo humano, as razões inconfessáveis e conseguem expor isso de modo magistral.

Se você é anarquista, você não esta agindo de forma incoerente bebendo uma coca cola que é uma marca registrada do capitalismo selvagem que explora o proletariado?‎

Sim. Estou sendo incoerente.

você é petista ou um tucano ou um ignorante igual a mim?‎

Não sou filiado a nenhum partido político porque os partidos no Brasil não possuem nenhum programa que verdadeiramente sigam, construído em torno de concepções políticas filosoficamente estabelecidas. Algum que o tenha, eu não concordo. Todavia não sou um analfabeto político. Voto de modo consciente, considerando a pessoa do candidato. Mas não considero que o fato de ter dado meu voto a alguém me alinhe incondicionalmente a essa pessoa. Se ela agir em desacordo com o que eu esperava dela, ou se defender posições que eu não defenda, discordarei dela em o menor problema.

Se o tempo for quantizado, isso implica em quê?‎

Implica que a passagem do tempo se dá aos saltos e não contínua e suavemente. Ou seja, os estados sucessivos do Universo, cujas mudanças geram a passagem do tempo, alteram-se bruscamente em degraus sem situações intermediárias. Se o quantum de tempo for o "chronon" de Caldirola, isto é, cerca de 6.27×10^−24 segundos, o Universo se modificaria aos saltos e cada salto representaria a passagem de um tempo desses, ou seja, de 627 oitihonésimos de segundo. Da mesma forma uma quantização do espaço significaria que as mudanças de posição de algum ponto só poderiam ocorrer aos saltos do quantum de espaço, que seria, no caso, 1,88x10^-15 metros (correpondente ao Chronon). Não há, contudo, nenhuma evidência conclusiva de que nem o tempo nem o espaço sejam quantizados.

Um tempo de Planck é o menor tempo possível de se existir?‎

Não. O tempo de Planck é um valor construído com as constantes fundamentais da natureza. A busca de um "quantum" de tempo, que seria o menor intervalo possível, conduz a um valor, até, maior do que o tempo de Planck. Mas não é estabelecido que nem o tempo nem o espaço sejam quantizados. Pode ser que sim, mas pode ser que não. Veja isto:
http://en.wikipedia.org/wiki/Chronon
http://en.wikipedia.org/wiki/Planck_time
http://en.wikipedia.org/wiki/Gravastar

E no caso dos documentários em geral. Os de hoje gravados, não parecem desviar o foco do tema, incluindo outros só para chamar atenção dos espectadores? Em mimha visão, apesar dos documentários atuais terem uma melhor visão cientifica e tecnológica, meio que virou comércio, os assuntos principais.‎

De fato acontece dos documentários estarem enviesando para um aspecto apelativo de audiência, levando os temas para o lado do que seja mais chamativo e, não necessariamente, para o que seja mais significativo. Isso também acontece com a forma de abordagem, que acaba ficando sensacionalista demais, perdendo a isenção científica e histórica que sempre indica um modo parcimonioso de tratar a temática, isto é, evitando qualquer apelo emocional. Um pena... Mesmo assim, é melhor que documentários sejam assistidos do que outros tipos de programas televisivos, como os de auditório e, mesmo, novelas. Sem mencionar os jornais que quase só abordam crimes e catástrofes.

A TV Cultura é ótima mesmo. Os programas infantis que assisti eram sensacionais, educativos. Agora, assisto a filmes de vários países da América Latina na Cultura e eles são muito bons. Também passam documentários. Você assiste eles também?‎

Assisto, mas pouco, porque trabalho umas doze horas por dia, durmo umas seis ou sete e ainda quero ler e escrever. Então não sobra tempo.

Haveria algo no universo que não estivesse sujeito ao tempo? A informação, por exemplo?‎

Tudo que exiba uma conservação é imunde ao tempo. Este é o caso da energia de um sistema isolado, da quantidade de movimento, do momento angular, da carga elétrica e outras grandezas que meçam atributos de sistemas. Grandezas há, contudo, que não exibem conservação, como a entropia e o volume, que são extensivas, bem como as intensivas, como temperatura e densidade. Informação pode ser quantificada e é extensiva. Como é uma função da probabilidade e esta é conservada na Física Quântica, seria conservada. Todavia, em relatividade geral, ela não o é. Veja isto:
http://en.wikipedia.org/wiki/Quantities_of_information
http://en.wikipedia.org/wiki/Information_theory
http://en.wikipedia.org/wiki/Philosophy_of_information
http://en.wikipedia.org/wiki/Probability_current
http://en.wikipedia.org/wiki/Black_hole_information_paradox

Tu não assiste televisão (se eu não estou enganado) não é professor? Mesmo não assistindo - se for o caso - teins alguma visão e crítica sobre a Rede Globo, ou até mesmo as demais televisões brasileiras no geral. Deveriam ser diferentes?‎

Não gosto da programação das emissoras comerciais de televisão. Mas gosto da TV Cultura, por exemplo, bem como de outras educativas e de várias por assinatura, como a NatGeo, a Discovery. Quanto ao History Channel, não gosto de seus documentários sobre UFOS (OVNS), pois eles apresentam conjecturas como se fossem confirmadas. As redes comerciais são parciais, apresentando a visão de mundo compartilhada por seus donos, não mostrando todas as facetas da realidade que apresentam.

https://www.youtube.com/watch?v=VzT2ZIJX6AY O que acha desta composição musical de Nietzsche?‎

Nietzsche era um bom compositor. Mas exibe mais técnica do que arte. Em suma, uma demonstração de inteligência musical, mas não de criatividade musical. O mesmo se pode dizer das demais composições dele.

Ernesto, vc acredita na existência de métodos de estudos que possibilitem melhores êxitos em vestibulares e concursos públicos? Uma pessoa que estude por menos tempo e consiga se igualar ou sobressair em relação àquele que estudou por mais tempo? Algo relacionado a uma técnica de melhor aproveitamen‎

Não acho que técnicas sejam eficazes para o aprendizado. O que vale, para obter boa pontuação em qualquer exame é, de fato, saber o assunto. E se fica sabendo o assunto, em primeiro lugar, se se cria uma relação de gosto por ele. Não se aprende o que se detesta. A partir do gosto e do interesse, vai-se dedicando a querer saber mesmo, pra valer. Isso é que faz a diferença. A pessoa estuda muito porque é o que quer fazer. Quer saber muito aquilo. Então fica horas a fio estudando com todo o prazer. Então aprende.

Professor, o que seria um O.V.N.I. na sua opinião? Tecnologia humana?

OVNI significa "objeto voador não identificado". No momento que for identificado deixa de ser um ovni. Podem ser fenômenos meteorológicos, balões, artefatos voadores humanos ou, simplesmente, fraudes cinematográficas. Quanto a serem naves extra-terrestres, não conheço nenhuma comprovação disso.

Ernesto, por que os a maioria dos grandes pensadores, filósofos e cientistas são europeus ou descendentes de europeus? É cultura, genética ou coincidência? Por que?

Porque a civilização européia, fortemente influenciada pelas concepções greco-romanas, valorizou o exame especulativo da realidade. Mesmo com o retrocesso medieval, em que se colocou a fé acima da razão, tal espírito não foi totalmente vencido e ressurgiu. As civilizações orientais são mais práticas ou mais religiosas, não valorizando muito a investigação sem pré-concepções que caracteriza o fazer científico e filosófico. Trata-se, pois, de um viés cultural. Em suas origens gregas, não sei dizer o que determinou esse modo de encarar o mundo. Mas é bem diferente do judaico, do babilônico, do egípcio, do indiano, do chinês.

É possível alguém "comum", com inteligência dentro da média, e caso se esforce bastante, pelo tempo que for necessário, aprender pelo menos o básico-intermediário de tudo que tem vontade? Ou isso exige, sem dúvida, uma inteligente acima do "normal"?

Sim, claro que pode. Todo mundo tem alguma inteligência. Maior inteligência implica em mais facilidade de aprendizado, mas menor inteligência não implica em impossibilidade de aprendizado. Ele se dará com mais dificuldade e demora, mas acaba acontecendo.

Existe relação entre a Física quântica é a neurociência ? Em caso afirmativo, favor exemplificar.‎

Claro que sim, por um motivo reducionista, uma vez que os fenômenos biológicos são químicos, os fenômenos químicos são físicos e os fenômenos físicos são quânticos. Sem escapatória. Então o pensamento, as emoções, as volições e todos os fatos psíquicos só acontecem porque existem interações quânticas entre prótons e elétrons nos átomos. Senão eles não se dariam. Todavia as propostas de explicações, por exemplo, da consciência, em termos de ocorrência quânticas de larga escala, dentro do cérebro, como propõem Hameroff e Penrose não são confirmadas.
http://hypescience.com/mecanica-quantica-alma/

Você ja trabalhou em instituição em que o deputado federal Bonifacio Andrada é presidente, me conte qual era a sua função e como você se sentia em trabalhar para um representante do "povo" que é um capitali‎

Fui professor da Faculdade de Filosofia de Barbacena e de seu Colégio de Aplicação nos anos de 1972 a 1976. O deputado Andrada quase não aparecia por lá. Mas eu não via diferença em trabalhar para ele ou para outro qualquer. O que eu me preocupava era em ensinar bem ensinado.

É verdade que não posso saber se o mesmo vermelho que você enxerga é o que eu enxergo também?‎

Certamente que não, pois a sensação e a percepção são subjetivas. Mas podemos concordar sobre o matiz e o tom de qualquer cor, especificando-o corretamente, de modo que ao me descrever um tom e matiz, eu aponto a cor e você concorda. Só que o que eu percebo não há como saber que é o que você percebe. O mesmo pode se dizer dos sons, odores e sabores, por exemplo.

Ernesto, você sempre gostou de música clássica? Ou já se aventurou em outros estilos musicais?‎

Desde criança que gosto de música clássica, pois meus pais ouviam na vitrola e minha mãe tocava no piano. Com nove anos comecei a estudar piano e parei aos doze, para ter mais tempo para estudar. Mas também gostava, e ainda gosto, de samba tradicional, tango, jazz, bossa-nova, "easy music" e, até, música caipira de raiz. De rock eu gosto de alguns. Não gosto de axé, funk e música com muito batidão. Nem de sertanejo universitário.

O senhor acha que nós podemos estar vivendo em um tipo de "matrix" ? Como se fossemos USB's conectados a um computador?‎

Não acho não. Isso é uma conjectura ficcional.

Por que se fala que o sol tem oito minutos luz da terra se a galaxia esta se afastando?‎

Nossa galáxia não está se afastando de nós. Nós estamos dentro dela, girando com ela em torno do seu centro. O que está se afastando são as galáxias, umas das outras. Mas isso só se observa para galáxias distantes. As próximas têm o seu movimento próprio aleatório. Inclusive Andrômeda, que é a galáxia grande mais próxima, está em rota de colisão com a nossa e a colisão acontecerá dentro de 3500 milhões da anos. A luz do Sol, que está a 150 milhões de quilômetros de nós, leva uns oito minutos para nos atingir. O centro de nossa galáxia, a Via Láctea, está a cerca de 26.000 anos luz de nós. O diâmetro dela é de 100.000 anos luz. Veja isto:
http://pt.wikipedia.org/wiki/Sol
http://pt.wikipedia.org/wiki/Via_L%C3%A1ctea
http://pt.wikipedia.org/wiki/Big_Bang

Todo filósofo tem que seguir um raciocínio lógico?‎

Depende da ocasião. Se ele estiver conduzindo um raciocínio para demonstrar algum argumento, claro que ele tem que ser lógico. Mas se ele estiver elaborando uma proposta de explicação de algo, a partir de suas observações e reflexões, ele não precisa ser lógico. Nem um filósofo, nem um matemático, nem um cientista. Nesse momento ele precisa ser é intuitivo e deixar aflorar tudo que venha à mente sem restrição nem censura. Depois que ele chegou à conclusão do que acha que deva ser é que precisa buscar uma argumentação consistente para defender sua proposta. Então, para apresentá-la, precisa ser lógico.

Você se veste bem quando sai à rua? Por quê?

Sim. Porque acho que é uma forma de respeitar as outras pessoas. Além de também ser uma forma de demonstrar asseio, correção, cordialidade, distinção, apuro, elegância, nobreza, cortesia, seriedade e virtudes similares que se deve praticar para fazer a convivência humana em sociedade um fato agradável, prazeroso, benfazejo. Isso se aplica não só ao modo de se vestir, mas ao modo de se falar e de se comportar educadamente. Não só em público mas, inclusive, em particular. Nada justifica um modo de ser relaxado, prosaico, deselegante e, porque não dizer, porco e chulo. Ser uma pessoa igualitária não significa ser uma pessoa sem distinção nem educação.

Qual area de humanas o senhor acha mais interessante?‎

Filosofia, que é considerada uma disciplina das humanidades, mas em verdade não é. Filosofia paira acima de todos os conhecimentos de todas as áreas. Também gosto de História e Linguística. Mas não gosto de Direito, Administração, Economia.

Se no caso, o dia do grande juízo final existir, você nao tem medo de ir pro lago de enxofre? Ja que você é ateu!‎

Claro que não. Mesmo que Deus existisse, porque ele condenaria uma pessoa boa? Depois, já parou para pensar como é ridícula essa noção de inferno como um lugar de fogo ou de enxofre? Já pensou como é impossível haver um lugar assim por toda a eternidade se, com a expansão do Universo, haverá a ruptura das partículas elementares que formam a matéria que, então não existirá mais? Portanto não poderá haver fogo eterno, pois fogo é um gás incandescente e, portanto, matéria. Há tanta incoerência na concepção da existência de um céu e de um inferno que não se pode considerar que seja uma proposta válida. Da mesma forma a existência de uma alma, a ressurreição dos corpos no juízo final e tudo isso. O que inclui, também, a própria existência de Deus. Trata-se de algo inteiramente sem cabimento.

Também gosto de coca-cola. Mas prefiro a com açúcar. Tu prefere a zero por não conter açúcar ou por gosto?‎

Por não conter açúcar, já que sou diabético. E o chocolate que como é sem açúcar também.

foi feito um teste de QI em uma criança de 12 anos e deu 142. 4 anos depois ela fez de novo, tem como o resultado descer muito?‎

Os testes de QI têm que ser adequados a cada faixa etária e ao nível de escolaridade da pessoa. Se for aplicado o teste incorreto, o resultado será errado. Todavia o QI pode variar para mais ou para menos, em função de como a pessoa passou o tempo entre os testes. Quem exercitou a inteligência pode ter um aumento. Quem ficou sem desafios mentais, pode ter uma diminuição. Mas essas variações não são tão grandes, podendo chegar até, mais ou menos, 15%, se o prazo for de uns cinco anos.

No infarto, você fica tonto/desnorteado durante? O coração fica sempre muito acelerado, ou pode estar batendo normalmente?

Tive dois infartos e em nenhum deles fiquei tonto. O que senti foi uma grande ardência no peito, como uma queimadura, uma grande dormência no braço esquerdo e uma forte dor no estômago, como se tivesse levado um murro. Mas isso pode variar de pessoa para pessoa.

O que acha da obra de Diderot?‎

Diderot é um autor de quem ainda não li nada.

Quais são os seus vícios?

Ler, tomar coca-cola zero, comer chocolate, ouvir música clássica, responder neste ask...

Você acha adequado iniciar formação profissional na área de saúde mental e psiquiatria antes de formar-se em medicina ? Um conhecimento desleixado nas áreas gerais da medicina pode levar a um mal compreendimento do tratamento de enfermidades mentais ?

Claro que é. Para ser psiquiatra primeiro tem-se que ser médico.

Já deve ter dito isso aqui antes, mas você possui doutorado em física ?

Não. Possuo mestrado. Veja meu perfil. Ou meu currículo: http://www.ruckert.pro.br/blog/?page_id=6

Ernesto, vc é um Físico Teórico ou um Físico Experimental?

Teórico. Se bem que, como professor, também lecionei física experimental. Fui até coordenador do laboratório didático de eletricidade, magnetismo e ótica da UFJF.

“Como Diderot já disse, em O Sobrinho de Rameau, a pessoa que ensina a ciência não é a mesma que entende dela e a realiza com seriedade, pois a esta não sobra tempo para ensinar.” (Schopenhauer) - O que pensa?‎

Discordo. Acho que é perfeitamente possível que quem construa a ciência a ensine. Não para a Educação Básica, mas para quem for ser professor. Acho que é muito salutar que as universidades exijam que o professor também seja pesquisador e que o pesquisador também seja professor. Assim essas duas vivências se entrelaçam, o que é excelente. Mas é preciso que, como professor, o pesquisador também tenha didática. Porque para ensinar não só é preciso saber o que se vai ensinar mas também como se deve ensinar.

Professor, tenho 21 anos e abandonei a faculdade particular de Direito, no 3º semestre, para cursar História (Licenciatura e Bacharelado), área que me encanta e que consome muito tempo de minhas leituras, numa Universidade Federal. Sonho em lecionar com qualidade, dinâmica e muito amor. Sou louco?‎

Claro que não. Você é perfeitamente lúcido. Além de ser uma pessoa consciente da responsabilidade que é preciso ter em relação à atividade didática. Que bom que muitos fossem como você. Isso levaria o nível da qualidade do ensino em nossas escolas, especialmente públicas a patamares desejáveis. Além de criar condições para que bons professores possam reivindicar bons salários para o magistério. Não digo salários razoáveis mas salários atrativos. Que façam as pessoas preferirem ser professores do que ser médicos, engenheiros ou advogados, porque se ganha mais. Que permitam uma seleção das melhores capacidades exatamente para serem os professores. Isso é que tem que acontecer para que possamos atingir um nível cultural, científico e tecnológico elevado, como precisa ser. Para tal é preciso que pioneiros se disponham a ser ótimos professores ganhando mal mesmo, a fim de conseguir que o magistério seja muito bem remunerado como tem que ser. Não estou dizendo bem remunerado. Estou dizendo muito bem remunerado.

Professor, todos os testes de QI que fiz deram em torno de 135 - 137. Sempre entre esses números. Existe algum site onde eu possa verificar com mais exatidão? Onde fez teus testes?‎

Testes de QI não são exatos. O jeito é fazer vários e tirar a média. Há muitos sites na internet que os apresentam. Consulte o site http://www.cpsimoes.net/

Como julga o suicídio?‎

Perfeitamente legítimo e inteiramente não recomendado.

Ernesto, vc acredita na Teoria dos Astronautas Antigos ou na Teoria do Paleocontato de Carl Sagan? Por que?

Não acredito. Considero que a possibilidade de algum contato com seres alienígenas inteligentes seja extremamente difícil, mesmo que não impossível. Provavelmente só haveria três planetas com vida inteligente, em média, por galáxia, o que colocaria os da nossa a dezenas de milhares de anos-luz de distância, inviabilizando o contato. Por outro lado, não há nenhuma evidência de que esses contatos tenham havido e nem de que a evolução cultural e tecnológica da humanidade seja devida a alguma interveniência alienígena.

É... a foto de fundo da sua ask... ela foi tirada há quantos milhões ou bilhões de anos-luz da via láctea?‎

Trata-se da estrela Wolf Rayet HD 50896, situada a cinco mil anos luz da Terra, dentro desta galáxia. Veja isto:
http://www.sci-news.com/astronomy/article00695.html
Note que o diâmetro desta galáxia é pouco mais de cem mil anos luz e a vizinha Andrômeda está a dois milhões e meio de anos luz. 8 dias atrás

Pertencer a Mensa realmente diz algo sobre a inteligência de alguém? Diversos que se gabam disso, mas analisando suas visões de mundo parecem levianos em vários aspectos. Causam a impressão de serem eruditos, mas na verdade apenas pontuaram X num teste que avalia de forma limitadíssima o intelecto.‎

A participação na Mensa exige um QI mínimo em torno de 132 ou 136. O QI é um indicativo da inteligência mas não é cabal. Ou seja, é possível que pessoas mais inteligentes exibam QI menores do que pessoas menos inteligentes. Por outro lado, há algo mais importante do que a inteligência que é o caráter. O que eu percebi, pelo menos entre os mensanos brasileiros, é que eles são muito vaidosos e isso é uma falha de caráter. Quem seja inteligente não tem mérito por ser assim, pois, em geral, não é por seu esforço e sim por uma coincidência genética. Então essa pessoa tem que colocar sua inteligência a serviço do mundo sem gabolice.

Por que cada cor nos afeta de uma forma e causa uma impressão especifica? Uma pessoa vestindo uma cor causa uma impressão X, mas com outra cor causa uma impressão Y. A simples variação na cor de um objeto pode me fazer admira-lo ou ser indiferente. Seja uma veste, uma guitarra, uma flor. Por que?‎

Há estudos psicológicos sobre isso, mas não estou inteirado. Sei que o efeito existe, mas não como opera. Isso se pode achar na internet. É só investigar bem.

Concorda? https://www.youtube.com/watch?v=j0WCvNyHaqE&feature=youtube_gdata_player‎

Plenamente. Muitos que protestam contra a corrupção do governo usam programas piratas, estacionam em lugares proibidos, colam nas provas e assim por diante. Portanto a corrupção é um mal endêmico introjetado na sociedade brasileira de modo enraizado e difícil de ser extirpado. Só a educação conseguirá isso, mas ao longo de várias gerações (isto é, séculos). Conscientizar todo mundo de que se a mentira der lucro e a verdade prejuízo se deve querer levar o prejuízo para atender à verdade. É preciso começar a agir assim nas mínimas coisas. No dia a dia. Senão não se tem moral para cobrar honestidade de político, juiz ou empresário nenhum.

Concorda que a unica unidade de medida que se pode ter certeza é o tempo? Abraços Ernesto, te acompanho a 2 anos aproximadamente.‎

Não. Todas as unidades de medida possuem incertezas, inclusive as de tempo. Elas são definidas do modo que se possa ter a maior precisão possível, mas não se tem exatidão. Para nenhuma.

Professor Ernesto, consta do livro O Universo que, segundo o cientista Paul M. Dirac, existe a possibilidade de as propriedades da matéria estarem mudando, motivo pelo qual a física que conhecemos hoje pode não ser a de ontem. O que acha?‎

A questão é concernente aos valores das constantes, como a velocidade da luz, a constante de Planck, a constante de Boltzmann, a constante da gravitação universal e outras. De fato, pode ser que os valores delas não sejam constantes e sim uma função do tempo. Assim o sendo, as equações que as envolvem teriam que considerá-las não como constantes, mas como campos e se teria que achar o modo como seus valores variam. Isso, de fato, modifica a Física nos eventos que envolvam decursos longos de tempo, da ordem de bilhões de anos. Todavia ainda não se sabe se é isso mesmo e, se for, como se daria a variação delas. O futuro das pesquisas poderá levar a esse tipo de conhecimento.

https://www.youtube.com/watch?v=AXhRmv1mrs4&t=10 Quais frequências você foi capaz de ouvir?

de 90Hz a 11kHz. Mas o ambiente não estava completamente silencioso.

A matemática é exata? Por quê?‎

Depende. A maior parte da matemática, que se refere às relações entre grandezas idealizadas, é exata. Todavia, quando essas grandezas forem expressas por valores medidos, então os resultados deixam de ser exatos, uma vez que toda medida possui alguma aproximação.

por que o carro e um avião protege os passageiros de raios ?‎

Porque eles são condutores e envolvem quem está dentro por todos os lados, criando uma blindagem eletrostática ou uma "Gaiola de Faraday". O fato de uma superfície condutora envolvente não permitir campos elétricos em seu interior é decorrente do fato da força elétrica variar com o inverso do quadrado da distância e, na geometria euclideana, as áreas variarem com o quadrado da distância, causando uma compensação. Veja isto:
http://pt.wikipedia.org/wiki/Gaiola_de_Faraday

Existe diferença entre Céu e Inferno?

Nenhuma, uma vez que nem um nem outro existem, Ou seja, são um conjunto vazio e todos os vazios são iguais, isto é, só há um conjunto vazio. São apenas nomes diferentes para a mesma coisa, isto é, nada.

Com quais outros sistemas operacionais além do Windows você tem experiência?‎

Já usei o Linux Ubuntu, mas não com muita frequência. Antigamente eu usava o DOS, o que fiz ao longo de muitos anos. Também já usei o UNIX em uma estação de trabalho SUM que havia no Departamento de Física, mas poucas vezes. No Main Frame o sistema era o DM2. Já não me lembro como usar.

O que você acha das pessoas que ficam pedindo intervenção militar nos protestos? É algo que me assusta muito, sinceramente k‎

É que elas não sabem o que seja viver sem liberdade. Mesmo sendo radicalmente contra essa corrupção que existe por aí, jamais considero que a solução seja a volta dos militares em algum governo de força. A democracia é que tem que prevalecer. E a Dilma foi eleita democraticamente, mesmo que sem o meu voto. A maior virtude democrática, contudo, é os vencidos aceitarem a derrota e defenderem o direito dos vencedores governarem. Também não acho que se configura uma condição de "impeachment" da Dilma. Mau governo não é motivo para isso. Muito menos governo que desagrade.

Nunca aprendeu a programar em Assembly? Pretende?

Não. Já programei em Fortran no tempo dos "Main Frames" IBM 1130 e IBM 360. Agora não tenho mais interesse em aprender novas linguagens de programação. Prefiro estudar Filosofia e Cosmologia.

http://g1.globo.com/jornal-nacional/noticia/2015/03/aos-10-anos-menina-ja-leu-mais-de-400-livros-e-monta-biblioteca-em-casa.html tu era assim Prof?‎

Nem tanto mas, mais ou menos. Meu pai já tinha uma boa biblioteca, mas eu ganhei uma estante para o meu quarto desde o curso primário e já ia arranjando livros. Também lia muitas revistas de quadrinhos, como "O Pato Donald" e "Mickey". Além de "Seleções". Ainda tenho os livros de minha infância.

Eu pensava que inventar era criar o que ainda não foi inventando. Você acha que inventar o que já foi inventando é inventar?‎

Do ponto de vista de quem inventa, sim, se ele não conhecer que já foi inventado. Do ponto de vista do mundo não, pois já foi inventado. Mas pode não ser conhecido em todo o mundo. Isso acontecia muito antigamente, quando as comunicações eram precárias. Muitas invenções foram feitas separadamente em lugares diferentes, como barcos, a roda, o fogo, o tijolo, os vasos de cerâmica, a forja de metais, o vidro, o tecido (pano), a curtição do couro, as sandálias, os sapatos, o martelo, a faca, o arado e muito mais.

O que vocês acham que as escolas não ensinam, mas deveriam ensinar?

Muita coisa: culinária, horticultura, jardinocultura, direção de veículos, hidráulica, eletrotécnica, mecânica de autos, serviço de pedreiro, corte e costura, contabilidade, informática, pintura, música, canto, teatro, escultura, retórica, psicologia, vavar e passar roupa, cerâmica, natação e muita coisa útil para a vida. Não tudo para todo mundo, mas como opções. A escola teria que ser de tempo integral, com refeição na escola preparada pelos alunos com vegetais plantados por eles. Eles teriam que limpar as salas e assim por diante. E só deveriam existir escolas públicas, governamentais ou comunitárias. Melhor é que fossem todas comunitárias, financiadas e controladas pelo governo. Enquanto houver governo no mundo.

O que fazer quando estamos apaixonados por alguém que só vê relação de amizade em nós?

Continuar amando mas mantendo um relacionamento só amistoso. Isso é melhor do que evitar amar ou se afastar da pessoa. Mas a pessoa deve saber que é amada. No entanto não se pode forçar um relacionamento amoroso não correspondido, pois isso levaria a pessoa a interromper a amizade. Enquanto isso, agir no sentido de conquistar o seu amor, sem pressão. Paralelamente é bom, também, se abrir para a possibilidade de amar, também, a outra pessoa, que possa corresponder e atar um relacionamento. Desde que essa outra pessoa saiba do amor pela primeira. Se a primeira já tiver um relacionamento com alguém, quem sabe ela não se abre para um relacionamento plural, se aquele alguém também concordar. Essas possibilidades são a melhor forma de conciliar a felicidade com amores múltiplos, pois não considera que os relacionamentos que esses amores podem levar tenham que ser exclusivistas. Certamente que o amor não é necessariamente exclusivista. A questão é que, havendo amor e desejo, quer-se que isso se realize em uma relação estabelecida. Mas as pessoas, por enquanto, resistem a admitir que essas relações possam ser múltiplas com toda a intensidade e sinceridade das relações biunívocas. Amor, quanto mais melhor. E sofrer porque o amor não é realizado é muito ruim. Então, que se realizem todos os amores. Não há nada nisso que fira a ética. Como já aconteceu com a imoralidade da perda de virgindade fora do casamento para as mulheres, essa é uma concepção da moral que precisa ser alterada. Isso, em nada, obra contra o valor dos laços familiares, como alguns consideram. Se se admite a pluralidade relacional sequencial, por que não admitir a paralela?

O que fazer quando fomos traída?‎

Há três alternativas. A primeira é romper com quem você se relaciona. A segunda é não romper, condicionado ao fato de que a pessoa, doravante, se relacione exclusivamente com você. A terceira é admitir a possibilidade do relacionamento plural e os três continuarem se relacionando. Isso vai depender de suas concepções de vida. Não acho a segunda interessante, pois se a pessoa teve um relacionamento paralelo, não revelado a você, é porque ela considera que isso seja uma possibilidade e, portanto, não haverá garantia de que não se repita. A primeira pode ser muito traumática, se você, de fato, ama a pessoa. A terceira, para mim, é a mais tranquila e mais favorável à felicidade de todos os envolvidos. O único problema é que a sociedade considera que seja uma imoralidade. Todavia, no caso, a moral é que está errada, pois não se fere, em absoluto, ética nenhuma, uma vez que os envolvidos estão de acordo e todos felizes. Mas há quem não consiga arrostar a sociedade e fique incomodada de agir de forma inaceitável por muita gente, especialmente se nessa gente se incluir pessoas queridas, como os familiares. Mas é o que recomendo.

Ernesto, quero ser professor de História, pois amo os estudos e livros que, em geral, tratam do assunto. O recomendável, inicialmente, seria cursar a Licenciatura ou o Bacharelado? Pretendo fazer as duas coisas, mas por qual dos dois eu começo, tendo em vista meu sonho de ser professor?‎

Sempre recomendo que se faça tanto o bacharelado quanto a licenciatura. Como a licenciatura é mais fácil e já dá uma profissão, seria melhor fazê-la primeiro. Mas não deixe de fazer o bacharelado, pois um licenciado sem bacharelado é um profissional mal formado. O ideal, para ser um excelente professor (e nunca se deve desejar menos do que a excelência) é, também, fazer mestrado e doutorado e, mesmo, pós-doutorado. Então, além de ser um professor, de qualquer nível, pode-se ser um historiador mesmo. Note que um doutor em História pode, perfeitamente, ser um professor do Ensino Médio ou do Fundamental. Não há demérito nenhum nisso, como alguns consideram. E um professor de pós-graduação precisa, também, ter um domínio muito bom de didática e psicologia da aprendizagem, que se vê na licenciatura. Não há desculpa para um professor do terceiro ou quarto grau não ter didática.

Mandarei uma pergunta apenas para reflexão, não se ofenda. Abaixo o senhor afirmou que viagem astral não existe, que é bobagem. Por favor, vc poderia demonstrar essa proposição?

O que é preciso não e demonstrar que não exista e sim demonstrar que exista. Uma vez que se trata de ocorrência anômala em relação ao comportamento comum, a suposição básica é de que seja algum tipo de equívoco de interpretação ou, mesmo, de alguma fraude. Só se poderá aceitar que seja algo legítimo caso se mostre resistente a todas as tentativas de refutação, por meio de experimentos controlados por pessoal inteiramente cético. O que vejo a respeito de qualquer dita manifestação espiritualista é que ocorrem em situações sob o controle de quem pretende mostrar que sejam verídicas, o que compromete totalmente a credibilidade. Além do mais, viagem astral é uma ocorrência que requer que se admita que a mente possua alguma componente não física e que se possa "ver" sem um sensor de fótons, como é a retina do olho, "ouvir" sem um sensor de vibrações como é o ouvido interno e assim por diante. Tal tipo de coisa é algo que não posso admitir de modo nenhum. A mente é uma ocorrência que se dá em razão da extrema complexidade anatômica e fisiológica do cérebro e seus anexos e não tem como existir sem o substrato físico-biológico que a suporta.

A cada bela impressão que causamos, conquistamos um inimigo. Para ser popular é indispensável ser medíocre. (Oscar Wilde) - O que pensa?‎

Infelizmente o grosso da população tem um nível cultural, uma intelectualidade, uma sensibilidade artística e um conhecimento científico e filosófico elementares, de modo que acabam não apreciando as manifestações mais refinadas disso tudo. Como reação, em geral, ao invés de se esforçarem para conseguir compreender e apreciar tais expressões, acabam sendo refratárias e hostis a elas. Trata-se do reverso da medalha dos eruditos que desprezam as manifestações da cultura popular. Ambas atitudes reveladoras de um espírito mesquinho. Assim, para agradar a muitas pessoas avessas a algum modo de ser mais intelectualizado, mercê se faz despir-se da intelectualidade e se apresentar como não intelectualizado. Considero que isso não seja algo bom. Claro que abomino o pernosticismo ou qualquer tipo de gabolice. Todavia considero que a pessoa inteligente e culta possa se mostrar acessível a quem não o seja sem abdicar de sua cultura e inteligência. Não é indispensável ser medíocre não (ou fingir que o seja). Por outro lado, não vejo porque ser popular se configure em algo desejável. O importante é ser autêntico. Mas também é importante que essa autenticidade seja reveladora de um bom caráter e de uma personalidade acessível e amistosa.

Professor, qual dica o senhor dá pra quem quer fazer ITA?‎

Ser uma pessoa fissurada por física e matemática, que a vida toda adorou estudar isso e que tem um imenso cabedal de conhecimentos a respeito, sendo capaz de encarar os mais difíceis problemas desses assuntos com toda a tranquilidade. Então vai poder estudar pra valer, com todo o prazer, e ficar extremamente cobra na área. A pessoa para quem estudar seja um sacrifício e não curta física e matemática não vai conseguir estudar o tanto que precisa para passar no vestibular do ITA. Preenchida essa condição, há que se inteirar das questões dos anos anteriores e se debruçar para conseguir resolvê-las, estudando com profundidade o que for preciso. Em geral o nível da maioria dos cursinhos é baixo para o ITA. Tem que ver os mais puxados mesmo. O Poliedro é muito bom:
http://www.sistemapoliedro.com.br/ita/index.shtml

Levy Fidelix é condenado a pagar R$1 milhão por declarações homofóbicas (dinheiro que teoricamente será revertido em ações de promoção de igualdade da população LGBT). O que pensa sobre isso?

Corretíssimo!

Você está atribuindo caráter e personalidade a números? É isso mesmo que eu li?‎

Trata-se apenas de uma ilação e não algo real. É só a impressão que eles me provocam. Números não têm personalidade. Da mesma forma que cores, sons e aromas podem evocar diferentes estados mentais, números também o podem fazer. Como, também, movimentos, como em uma dança.

Professor, como eu posso tornar inteligente?

Oferecendo muitos desafios à sua mente. Complicando a sua vida. Fazendo tudo do modo mais difícil e sempre variado. Nunca fazer nada por hábito, mas sempre pensadamente. Usando sentidos variados para as atividades normais. Procure na internet pela palavra "neuróbica" e faça os exercícios recomendados. Leia os livros do Pierluigi Piazzi e os siga. Visite o site do Carlos Paula Simões.

E agora, o que pretende fazer?

Continuar respirando.

quinta-feira, 26 de março de 2015

Ernesto, rs, e você conta 49 vezes? Isso já não é suficientemente incômodo? Também tenho meus vícios numéricos! Eu salto, depois dois. Ex: 0, 3, 5, 8, 10, 13, etc. São sempre meus números escolhidos. Há exceções, pois gosto dos números 7 e 21.‎

49 é uma forma de dizer que tenho muita paciência e muita perseverança. Mas não muitíssima. Razoável seria sete. Muita 49 e muitíssima 490. É para se ter uma ideia do quanto resisto e insisto. Quanto aos números gosto mais dos ímpares e, dentre eles, os primos. Como 7, 11, 17, 19, 23, 29. Sempre que compro algo, compro essas quantidades. Acho os números primos serenos e altivos, ensimesmados e introspectivos, firmes e bravos. Mas não pernósticos nem soberbos. Afáveis com nobreza. Bondosos com energia, Justos sem frouxidão, mas compassivos e solidários.

O tempo não existe. É apenas uma convenção.‎

Existe sim. Respondi a isso alguns dias atrás. Pesquise.

Oi Ernesto, costuma ouvir Andre Rieu?‎

Sim, de vez em quando. Mas, de modo geral, prefiro outras interpretações. Acho as dele muito melosas. Todavia agradáveis, sem dúvida. Prefiro um Karajan, ou Metha, um Barenboin, um Böhn, um Bernstein, um Abbado, um Osawa, um Marriner, um Haitink, um Gergiev e outros grandes da batuta.

"Paciência tem limite. Até o cara mais pilantra sabe disso." Até onde vai a sua paciência? Consegues você lidar com tudo e todos na maior calmaria?

Até 49. É o meu limite. Tanto para paciência quanto para persistência. Tento 49 vezes. Depois desisto. Também aguento chateação insistente até 49 repetições. Depois dou um esbregue avassalador. Porque 49 é o quadrado de sete e sete é o meu número predileto que significa "muito".

Vale a pena ler "O Capital" e "Manifesto do Partido Comunista", do Marx? Existe algo nesses livros que você não concorda?‎

"O capital" é um livro difícil e sua leitura é penosa mesmo. Só já li resenhas sobre ele. O "manifesto" é uma obra curta e de fácil leitura. Não chega a ser penosa. É bom lê-lo. O que de fundamental eu não concordo é que seja a economia o motor exclusivo da evolução humana. Há outros fatores igualmente poderosos, como a religião, a vaidade, a busca da felicidade e outros. Também não concordo com a luta de classes. Acho que o que se precisa é levar os mais pobres a se tornarem ricos também. E não acho que seja por meio de uma revolução que o comunismo possa se estabelecer e sim por meio de uma evolução. Ou seja, pelo "socialismo utópico".

Uma pessoa que está inventando algo que já foi inventado,por desconhecer a história, não está contribuindo para o conhecimento no campo de pesquisa onde atua, Ernesto. Imagine, por exemplo, que alguém resolva inventar hoje equações da teoria da relatividade. Isto não seria algo completamente inútil?‎

Sim, mas não significa que ela não está inventando. Isso já aconteceu muitas vezes, inclusive pelo famoso matemático indiano Ramanujan. Vários assuntos foram descobertos independentemente por mais de uma pessoa.

O que é a fumaça? Nem sempre pode se dizer que "Onde há fumaça, há fogo", correto?‎

Fumaça é uma suspensão de sólido em gás, quando resultante de um processo de combustão incompleto. Portanto é sempre proveniente de algum fogo, pois o fogo são os gases produtos da reação de combustão enquanto sua temperatura está tão alta que ficam luminosos. Numa combustão completa não se produz fumaça e o fogo é feito só de água e gás carbônico (isso mesmo, água luminosa). Veja isto:
http://pt.wikipedia.org/wiki/Fumo
http://pt.wikipedia.org/wiki/Fogo

que eles não sabem da nossa existência‎

Claro! Baseado em que você afirmaria que sabem? Ou mesmo garantir que existam? Só se pode conjecturar que existam. Não há comprovação nenhuma.

Por que os E.T. não nos dão informações sobre sua origem e objetivos, não querem compartilhar conosco os aspectos técnicos de seus procedimentos e não desejam ser descobertos e muito menos interrompidos no que estão fazendo? Qual é a razão que os levam a fazer isso e quando esse processo vai parar?

Extra-terrestres nem sabem que existimos.

Você não é esquerdista nem aqui nem em jupiter.Até porquê você mesmo admitiu que não gosta de rótulos.Te dou a dica de não se rotular mais como nada.‎

Quando eu digo que sou algo não estou dizendo que concordo plenamente com tudo o que aquilo representa mas que, de modo geral, me identifico com aquela ideia. Assim não vou deixar de dizer que sou um esquerdista, mesmo discordando de muito do que esquerdistas defendem. Porque a esquerda é a concepção que considera as desigualdades intoleráveis. Essa é a sua essência. Todavia discordo da estatização da economia que a esquerda preconiza. Sou a favor de minimização do estado. Não concordo, em absoluto, com o socialismo de estado, mas defendo o comunismo, isto é, a posse comum dos meios de produção. A posse pelas pessoas e não pelo estado. Ou seja, preconizo a abolição do trabalho assalariado e a transformação dos trabalhadores em sócios dos empreendimentos. Ao invés de acabar com a burguesia, acabar com o proletariado e transformar todos em burgueses, com a riqueza distribuída da forma mais igualitária possível. Note que distribuir o capital entre os trabalhadores não é ser capitalista. Capitalismo é a concepção de que o capital seja detido por parte da população e outra parte trabalhe, de modo assalariado, para a primeira parte. Se o capital for pulverizado por toda a sociedade, não haverá capitalismo. Até que se possa abolir totalmente o dinheiro e a propriedade. Outro ponto em que discordo de muitos esquerdistas é a via revolucionária de atingimento do comunismo, passando pela ditadura do proletariado. Isso é péssimo, como qualquer ditadura. E revoluções acarretam em manutenção de um poder pelos vencedores. O poder tem que ser abolido.

Recentemente um professor me disse, que tudo no universo é teoria. Ele está certo em sua afirmação?‎

Claro que não. O Universo existe independentemente de alguém fazer qualquer teoria sobre ele. Ele existe por conta própria. Não depende de inteligências e nem de consciências. Teorias são modelamentos de sua composição, estrutura e funcionamento que seres dotados de inteligência fazem para entendê-lo, compreendê-lo e controlá-lo, no que for possível. Se não houvesse seres inteligentes tudo seria como é, exceto pela inexistência deles. O que há no Universo são coisas e ocorrências. Teorias são abstrações formuladas por mentes e passadas entre elas.

Número de manisfestantes mostra que elite branca e opressora cresceu muito nos últimos dias e estão fazendo protesto nesse exato momento. Fonte: PT. kkkkkkkkkkkkkkkkkk.‎

Pelo que percebi, a maior parte dos que estão protestando não são pessoas da elite e não há apenas brancos. E eles estão protestando é contra os desmandos e contra a corrupção que enlameou o governo, especialmente (mas não somente) nos últimos mandatos do PT. Votei no PT muitas vezes até o Lula ser eleito. Depois não votei mais. Sou esquerdista mas não sou socialista. Muito menos marxista. Não estou satisfeito com esses governos. Mesmo achando que houve pontos em que fizeram coisas positivas. Também não gosto do PSDB e nem do PMDB, O PT se prostitui e eu o renego. Até que o anarco-comunismo se estabeleça no mundo sou um Social-Democrata. Mas o PSDB, apesar do nome, não é.

mas vc se lembra dos aliados será que me pode explicar um pouco disso?

Consultando na internet você consegue montar uma apostilas de mais de 500 páginas sobre o assunto e o dominar completamente. Sugiro começar pela Wikipedia (especialmente em inglês) e, depois, ir consultando os links que são citados lá. Já fiz isso para muitos assuntos e acabei me inteirando com bastante abrangência e profundidade sobre eles. Não busque nada resumido, pois não adianta. O bom é ficar sabendo do assunto de modo realmente extenso e profundo. Resumo é para você mesmo fazer, com suas próprias palavras, a partir do que você aprendeu não resumidamente. Outra alternativa é visitar uma biblioteca e lá estudar tudo sobre o tema.

Quando você olha pra sua cidade (Viçosa), você gosta do que vê?‎

Não. Acho Viçosa uma cidade muito suja, muito esburacada, com um trânsito muito confuso, muito verticalizada e concentrada no centro para o número de habitantes, muito pouco arborizada, de ruas muito estreitas, com poucos jardins e praças, muito violenta. O campus da UFV é uma exceção.

Não acha que rótulos políticos (esquerda, direita, anarco-capitalista, neo-marxista) são execráveis? Pense comigo: Uma pessoa pode simpatizar com ideias separadas de cada "hemisfério" sem necessariamente pertencer totalmente a ela. Quem rotula acaba discriminando baseado não na ideia, mas no rótulo.‎

Exatamente. Não gosto de rótulos. O que acontece comigo é justamente o que você disse. Concordo com parte do que cada um deles defende e discordo de outras partes. Assim sou execrado por capitalistas, por comunistas, por socialistas, por marxistas, por democratas, por anarquistas, por feministas, por machistas, por ateus, por crentes. Exatamente por aceitar, em parte, o que as concepções opostas de cada uma delas defendem. Mas não acho que deve me adequar a qualquer concepção. Inclusive filosófica. Como racionalismo, idealismo, empirismo, positivismo, criticismo, realismo, existencialismo, tomismo e por aí vai. Sou eu mesmo e, além disso, estou sempre mudando, à medida que estudo e aprendo. Mas não sou nada disso. E mais, acho um absurdo se enquadrar em qualquer concepção rígida. O ecletismo, tão repudiado por muitos, para mim, é a única concepção aceitável, pois não se enquadra em nenhuma.

Professor, Acredito que há uma energia inteligente por trás de tudo, sustentando a criação, que pode ser nominada Deus, natureza ou acaso. O que você tem a dizer sobre isto?‎

Não há não. Inteligência é uma decorrência da complexidade estrutural e funcional do sistema nervoso de alguns animais ditos "superiores". Não tem "energia" nenhuma associada. Só o seu funcionamento consome energia, propiciada pelos alimentos. Energia é o atributo dos sistemas físicos que os possibilita realizar algo, de qualquer ordem. A única coisa que fornece energia aos seres vivos é o alimento, no caso dos animais e a luz solar, no caso dos vegetais. Deus não existe. natureza é só um conceito relativo ao conjunto de tudo que não seja artificial, isto é, produzido por seres inteligentes, como os humanos e outros. E o acaso é só a qualidade de algo que tenha ocorrido sem que tenha sido provocado intencionalmente por nada. O acaso não é a causa de coisa alguma. O que se dá por acaso não tem causa.

Uma pessoa que sequer conhece a fundo a História da Filosofia, seria capaz de filosofar? Saul Kripke é um filósofo vivo, talvez um dos mais proeminantes deste século, e ele apenas possui um BA em Matemática. Filósofos de verdade são verdadeiros gênios, isto é um fato, por isso há tantos intérpretes.‎

Claro que para filosofar não é preciso conhecer a história da Filosofia. Mas se conhecer ajuda, desde que a pessoa não se prenda a isso, isto é, que não fique preocupada em ficar conferindo se o que está propondo já foi proposto por alguém e por quem. Isso é bobeira. Você pode inventar o que já foi inventado e estar inventando, pois você não sabe que já foi inventado. O que você não pode é pretender ter direito nenhum sobre o que cria ou inventa. Tudo o que se descobre é para ser doado ao mundo, sem pretensão de propriedade. Especialmente em ciência e filosofia. E ciência e filosofia não requerem nenhuma permissão legal para serem feitas. Se um advogado descobrir uma nova lei da Física, está descoberta e pronto. E se algum agricultor descobrir alguma relação filosófica ou propor algum sistema consistente, está feito, e pronto. Essa reserva de mercado do conhecimento a quem possua alguma formação formal a respeito é ridícula.

http://ask.fm/wolfedler/answer/125043639325 As vezes eu pensava em cursar Filosofia quando estava no ensino médio, mas não vale a pena. Nem no Brasil, nem em lugar nenhum. Só se aprende o que os filósofos disseram. E há professores sectarizados que não aceitam contestações de seus prediletos.‎

Essa é outra triste verdade. Esses professores parecem fundamentalistas religiosos, muitas vezes em relação a Marx, Nietzsche ou outros. Esquecem que a principal virtude do filósofo é ser um livre pensador. Ninguém é dono absoluto da verdade. Pode-se concordar na maior parte com o que alguém diga e discordar em outra parte. Por outro lado, Filosofia, diversamente de Medicina, por exemplo, é um assunto que se pode aprender por diletantismo. Mas os cursos de Filosofia deveriam (e então estariam sendo, de fato, cursos de Filosofia) desenvolver nos alunos o traquejo de filosofar. Deveriam levantar temas e promover discussões e formulações de hipóteses explicativas diversas das existentes. Bem como promover um debate entre as diferentes correntes de opinião sobre certo tema para ver quais deveriam ser rejeitadas e qual seria a correta. Rejeitadas que fossem todas, propor a que deveria ser correta. Isso é que é filosofar e o que deveria ser feito nos cursos. Do mesmo modo que um curso de música põe o aluno a tocar o instrumento, a compor, a reger a orquestra. E um curso de belas artes a pintar, a esculpir. Da forma como são conduzidos, os cursos de filosofia não fazem falta.

Qual a sua posição política?‎

Sou um anarco-comunista (anarco-posição política; comunista-posição econômica). Todavia, como isso não vai se dar em breve, até se chegar lá sou um Social-Democrata. Situação que não existe no espectro político partidário brasileiro. Por isso não me filio a partido nenhum. Pois não sou nem socialista, nem capitalista. Além do que nenhum dos partidos políticos brasileiros age em conformidade com sua plataforma.

"É muito importante que pessoas de outras áreas critiquem cada modalidade de conhecimento e atividade humana. Senão haveria uma ditadura corporativa extremamente nociva..." Mas não acha que diversos profissionais tem uma ideia de que apenas quem pertence a aquela classe pode falar sobre? Por que?‎

Sim, porque eles não querem ser criticados. Há um "espírito de classe", que protege internamente seus integrantes contra críticas externas. Isso é péssimo, porque não obriga os profissionais de cada área a serem muito zelosos de sua atuação criteriosa e competente, como precisaria ser. Eu, por exemplo, gosto muito de ser criticado e, mesmo, admoestado sempre que cometo enganos. Assim eu me aperfeiçoo. Ora, principalmente considerando que é a sociedade que mantém as escolas públicas e que grande parte dos profissionais nelas são formados, todos eles devem à sociedade a retribuição de prestarem um serviço de ótima qualidade. Assim têm que ser abertos a críticas, venham de onde vierem. Se não forem procedentes, têm que saber argumentar em defesa própria, a razão da improcedência.

Tenho a impressão que boa parte dos filósofos não tem curso formal. Você considera realmente desejável ter uma graduação? Tenho a impressão que isso limita a criatividade. O ambiente acadêmico parece fazer o aluno apenas absorver, mas não filosofar. Até os professores parecem meros repetidores.‎

Isso é verdade e é lamentável. Os cursos de Filosofia no Brasil, mesmo de pós-graduação, não formam filósofos e sim entendidos e comentadores de Filosofia. Terrível, mas é verdade. Eles não têm coragem de propor suas próprias idéias e ficam só estudando e comentando o que os outros disseram. Porque não são eles mesmos os filósofos? Os grandes filósofos foram grandes porque propuseram idéias completamente diferentes das que vigiam. Mesmo assim, é bom haver uma formação formal. Desde que se seja rebelde e não se acomode com essa situação. Além disso há um entendimento de que Filosofia seja uma disciplina da área das humanidades. Não é. Filosofia é uma meta-ciência, que abarca e se sobrepõe a todas as áreas de conhecimento: ciências humanas, ciências exatas, ciências biológicas, artes, negócios, política, seja o que for. Por isso o filósofo tem que entender de tudo. Principalmente matemática, física e biologia. Não é à toa que Platão mandou escrever no pórtico de sua Academia: "Não entre se não for geômetra". Mas isso não é só com a Filosofia. O Curso de Letras também não forma poetas e nem escritores. Só críticos literários (ou professores, no caso da licenciatura). No entanto o curso de música forma compositores e o de belas artes pintores e escultores.

Por que frequentemente vemos filósofos envolvidos em analisar outras ciências/pseudociências? Por exemplo, li uma critica do filósofo Onfray relacionada a psicanálise. Isso cabe aos filósofos? Onfray tem autoridade para isso? Como a filosofia pode servir de instrumento para criticar outras áreas?

Pode sim. A Filosofia é, justamente, a disciplina que cogita de analisar a propriedade ou impropriedade das demais áreas de conhecimento. Então o filósofo tem que se imiscuir e criticar tudo. Ciência, política, economia, artes. Seja o que for. Por isso é que ele precisa ser um polímata e ter uma formação completa, abrangente e de certa profundidade, não apenas humanista mas, também, científica e artística. Por outro lado, a concepção de que apenas os especialistas de certa área possuem competência para avaliá-la é totalmente despropositada. É muito importante que pessoas de outras áreas critiquem cada modalidade de conhecimento e atividade humana. Senão haveria uma ditadura corporativa extremamente nociva e totalmente insustentável. Políticos têm que criticar físicos. Físicos têm que criticar médicos. Médicos têm que criticar geógrafos. Geógrafos têm que criticar sacerdotes. E assim por diante.

Imposto de renda, por exemplo, é roubo, porque o trabalhador não pode usufruir do dinheiro do seu trabalho suado em totalidade.‎

Não é não. Aliás, o Imposto de Renda é o mais justo deles. O Imposto de Circulação de Mercadorias e o Imposto Sobre Produtos Industrializados é menos justo, pois incide igualmente sobre pobres e ricos. O Imposto de Renda, por ser escalonado, é mais justo. Só que sua alíquota deveria ter uma função crescente que levasse a valores maiores mais rapidamente, chegando até a, digamos, nove décimos da renda para grandes rendas e tendo um valor de isenção mais elevado. Eu sugiro uma função contínua, do tipo arco-tangente, ao invés de uma poligonal de segmentos retilíneos.

É verdade q o aquecimento global e o efeito estufa é uma farsa?

Existe efeito estufa. Sem ele a vida seria impossível na Terra, pois as noites seriam gélidas e os dias tórridos. E também existe aquecimento global nos dias atuais. O que é uma farsa é atribuir esse aquecimento, principalmente, a fatores antropogênicos. Estes também existem, mas não são os mais forçadores do aquecimento, que segue ciclos naturais de muito maior poder. Em verdade o aquecimento atual é uma subida de dente de uma serra descendente que nos levará, em poucos milhares de anos, a uma nova era glacial. Os ciclos naturais de temperatura da atmosfera da Terra existem e estão registrados em troncos de árvores milenares e outros registros, como a camada de gelo antártica. Eles mostram que a Terra já passou por fases de aquecimento como agora, mais intensas ainda, seguidas de fases de resfriamento. E há ciclos dentro de ciclos, dentro de ciclos, com períodos de décadas, séculos, milênios, dezenas de milênios, centenas de milênios, milhões de anos e maiores ainda. De fato, atualmente, as atividades humanas estão provocando um aquecimento maior. Mesmo sem elas, contudo, esse aquecimento estaria naturalmente acontecendo. isso não significa que não se devam envidar ingentes esforços para que o aquecimento seja menor.

Imposto é roubo.‎

O que se arrecada nos impostos pode ser roubado pelos governantes ou funcionários públicos. Mas isso não significa que o imposto, como instituição, seja um roubo, dentro do esquema crático da sociedade. Uma sociedade economicamente estruturada de forma monetária e possuindo governo que presta uma série de serviços, necessariamente vai ter que pagar o salário dos funcionários, bem como as despesas de manutenção da administração e os investimentos governamentais em saúde, educação, transportes, energia etc. De onde poderia vir esse dinheiro, senão dos impostos? A alternativa é uma sociedade sem estado e nem governo como, aliás, acho que deva ser. Mas isso não se consegue atingir de imediato, portanto, até que a evolução da civilização dispense os governos e o dinheiro, há que haver imposto e ele não é um roubo não, mesmo que possa haver quem roube o que se arrecada.

Ernesto, estou cursando neuroanatomia e tenho grande dificuldade em estudar pelo livro-texto (Neuroanatomia Funcional, do Ângelo Machado). Como se estuda um livro desses sem meramente decorar cada linha? Pergunto isso a você pois já o vi citando este livro para interessados em neurociências.‎

Estudar por um livro só é uma grande temeridade. Não se decora livro nenhum. Se um curso exigir tal tipo de aprendizado, então não é um bom curso. É preciso estudar o mesmo tema em vários livros e, então, fazer a síntese pessoal. Além do Ângelo Machado, recomendo os livros do Kandel, do Lent, do Gazzaniga e do Bear. Ao se ver o mesmo tema em várias abordagens o cérebro assimila o que há de comum a elas de uma forma natural.

Em uma pergunta antiga vc disse que um graduado em filosofia pode fazer mestrado em psicologia. Não percebo essa interface entre a filosofia e a psicologia. Em qual área da psicologia um filosofo estaria apto a fazer um mestrado? É fácil de achar uma instituição que permita isso?‎

A psicologia surgiu dentro do contexto da filosofia e, até hoje, ainda há uma linha filosófica na psicologia, que se estrutura a partir da introspecção e do raciocínio nela fundamentado e não na observação empírica e na experimentação científica. Portanto é viável que filósofos possam se dedicar à psicologia, especialmente na linha da "Filosofia da Mente". Caso queiram adentrar nos aspectos científicos da psicologia, então é preciso que cursem cadeiras científicas da graduação para fazerem o mestrado e o doutorado. Em verdade, considero que a psicologia deva ser uma especialização da medicina, baseada fortemente na neurologia e na psiquiatria. Ou que, pelo menos, inclua em seu currículo disciplinas médicas gerais (citologia, anatomia, fisiologia) e disciplinas médicas específicas (neurologia, neurociências, psiquiatria), como pré-requisitos das disciplinas de psicologia. E que os cursos de psicologia seja ecléticos, não se firmando em nenhuma corrente em particular, mas estudando todas elas. Isso levaria o curso a uns seis anos de duração, mas é assim mesmo que tem que ser. Acho que um psicólogo tem que ser um profissional tão bem preparado como um médico e o curso com o mesmo nível de dificuldade que a medicina. Dentro do curso, então, se veria, também, os aspectos filosóficos da psicologia, especialmente a "Filosofia da Mente".

"Constituição da Republica Federativa do Brasil (1988), art. 86, § 4º - O Presidente da República, na vigência de seu mandato, não pode ser responsabilizado por atos estranhos ao exercício de suas funções." Concorda com tal parágrafo?‎

Não. Acho um absurdo. Se ele ou ela cometer um crime passional, por exemplo, então não vai poder ser julgado? Isso é inadmissível. Um disparate.

Os impostos são necessários?‎

Em uma sociedade estruturada na forma de um estado que possua um governo que cuide do atendimento de vários serviços sociais e na qual a economia seja monetária, claro que o imposto é necessário, pois nada se faz sem dinheiro e o imposto é a forma do governo do estado arrecadar dinheiro. O imposto é mais justo do que as taxas, pois ele é cobrado de todos, mesmo dos que não usufruem dos benefícios do estado. Portanto promove uma distribuição de renda, especialmente quando suas alíquotas são progressivas em relação à renda.

eu tenho a impressão de que o budismo está muito mais próximo de ser uma filosofia de vida onde se busca uma postura ideal diante dos acontecimentos definida de acordo com um modo específico de compreendê-los,do que de uma religião,é incoerente desejar se alinhar a este ideal budista sendo ateu?‎

No budismo há um componente muito forte de filosofia de vida que pode, perfeitamente, ser adotada dentro de uma concepção atéia. Todavia há, também, um componente religioso, que é a concepção de karma e de sansara, que não é compatível com a concepção ateísta de que, além da não existir nenhum deus, também não existem espíritos, isto é, a realidade objetiva é uma realidade apenas física. Sem a concepção de alguma realidade não física, isto é, sobrenatural, mesmo não havendo deus, não é possível a realização da sansara e nem o karma tem nenhum significado.

O que acha do movimento estudantil JUNTOS?‎

Concordo com suas bandeiras e suas posturas.

http://ask.fm/wolfedler/answer/125038278429 A caracterização "fenomenológica" é no caso da escola originada por Husserl ou Hegel? Sei que você é eclético, eu também, não estou dizendo que é exatamente destes pensadores, estou usando apenas como referência.‎

Não. O que estou dizendo é que não se trata de uma conclusão obtida por raciocínio lógico e sim de modo empírico, isto é, a partir da observação do comportamento das diversas espécies animais, de níveis cada vez maior de complexidade neurológica. Algo é dito lógico quando decorre de raciocínio, é dito fenomenológico quando decorre da observação de fenômenos, é dito ontológico quando se refere à especificação do que venha a ser, é dito epistemológico quando cuida da forma de explicação de como ocorra e é dito teleológico quando se considera o propósito daquilo.

O que acha da frase "Todo preso é um preso político"? Concorda?

Claro que não. A maioria não é. São presos em razão de crimes cometidos sem razão política nenhuma. Há quem diga que sejam as condições sociais que fazem de alguém um criminoso e que as condições sociais são decorrentes de decisões políticas ao longo da história. Isso não é verdade. Mesmo que tais condições sejam agentes forçadores do crime, não são determinantes. Tanto é que há pessoas prejudicadas socialmente que não se tornam criminosas bem como criminosos nas camadas prósperas da sociedade.

Existe uma explicação lógica pro amor?‎

Existe explicação para o amor, mas não é lógica (nem ontológica, nem epistemológica, nem teleológica), mas sim fenomenológica. Amor é um complexo de ocorrências e situações que têm origem na pulsão biológica básica de propagar a vida, da qual vem o desejo sexual, bem como a proteção da prole. Isso leva os animais mais nervosamente complexos a desenvolverem um cuidado, também, com os parceiros sexuais e com o grupo a que pertencem. Na espécie humana, tais comportamentos instintivos se revelaram, pela evolução, em um sentimento de proteção, afeto, cuidado e carinho que, juntamente com o desejo, veio a ser o amor. Esse sentimento, contudo, também passou a se caracterizar por uma intelecção e um ato de vontade, isto é, uma conscientização do que seja, explicitada na mente, e uma resolução mental de agir em atendimento a tal sentimento.

Ernesto, qual é o objeto de estudo da ontologia? É a única área da filosofia que ainda não consegui entender claramente o propósito. Só acho explicações que soam obscuras pra quem não tem conhecimento: "É o estudo do Ser". Mas o que é o "ser"? De que se ocupa a ontologia?‎

Ontologia é a disciplina que busca responder, a respeito do que quer que seja, a pergunta: "o que é?". Isto é, caracterizar tudo, especificando as propriedades que possua de modo a que possa se dizer que seja o que é, bem como as propriedades acidentais, que possa ter ou não, sem que deixe de ser o que é. Ontologia é um capítulo da Metafísica, que abrange, além disso, outras considerações. Por exemplo a Metafísica estuda uma categorização da realidade. Dentre as categorias da realidade temos os seres. Para entender o que seja um ser, antes temos que saber o que seja um ente. Por ente se entende tudo o que possa existir, mesmo que não exista. Pode ser algo objetivo, fora das mentes, ou abstrato, não existente fora de mentes que o concebam. Mas não pode ser algo impossível, como um círculo quadrado. E também não é "tudo", pois se excluem as relações, os sentimentos, as emoções, as qualidades. as normas, as leis, as prescrições, os juízos, os raciocínios, as instituições e outros aspectos. A palavra que descreve tudo o que se possa conceber é "algo". Mas "algo" pode não ser um ente, nem uma coisa, nem um objeto. Um ente precisa ter uma "substância", isto é, precisa ser "feito" de algo, mesmo que seja um ente hipotético. O ente possui propriedades, também chamadas de qualidades ou atributos. Os atributos de um ente que o fazem ser o que é e não outro, se denomina sua essência. Mas um ente, mesmo que caracterizado por sua essência, pode não existir, de fato, na realidade. Quando um ente realmente existe, ele é dito um "ser". O existencialismo considera que o homem não possua uma essência, isto é, não possui propriedades que o caracterizem univocamente. Ele, para o existencialismo, apenas "existe". A questão da essência como necessária para caracterizar um ser, também ficou prejudicada com o advento da Física Quântica, em que se viu que as partículas elementares não possuem uma característica unívoca, podendo ser onda ou partícula dependendo da circunstância. Isso, ao lado das questões do indeterminismo e da incausalidade foram as principais implicações filosóficas da Física Quântica. Note que o indeterminismo e a incausalidade são questões metafísicas mas não ontológicas, pois não se referem ao que algo seja, mas sim a relações entre eventos experimentados por algo.

Você acha que humanos desenvolverem telecinesia é uma ideia absurda?

Sim, completamente. Não só humanos mas qualquer ser, do passado ou do futuro, daqui ou de qualquer lugar no Universo. Pois "telecinese" seria a capacidade de alterar o movimento de algum sistema físico por ação mental. E nenhuma mente é capaz de provocar interação física com algo fora dela. A não ser que emita um comando nervoso aos músculos do corpo em que se aloja para que esses músculos exerçam uma ação física sobre algum sistema e alterem seu movimento. Mesmo que essa ação física seja apertar botões de uma máquina, ou comandá-la pela voz. Não existe mudança de movimento sem interação física (isto é, aplicação de força). E pensamentos não aplicam força em nada.

Que postura obter diante de uma desilusão amorosa?‎

Conformar-se e se abrir para acolher um novo amor. Pois amor não se exige, não se compra e nem se vende. Amor só se dá e se ganha. Se se dá e não se ganha em retribuição, não há o que se reclamar. Todavia, é preciso que se trabalhe para obter a dádiva do amor de quem se deseja. Ou seja, conquistar essa benesse. Isso tem que ser feito com todo o desvelo, empenho e, principalmente, sinceridade. Qualquer conquista baseada em estratagemas urdidos de modo insincero é fadada ao fracasso, mesmo que, de início, possa ser eficaz. Porque não basta conquistar o amor. É preciso também preservar o amor, enquanto se ama. Portanto, jamais finja ser o que não se é. Jamais faça o papel de outrem que não de si mesmo. Se o modo próprio de ser não for bom para quem se deseja conquistar, desista da conquista ou mude o modo de ser de verdade, pra valer, passando a ser, sinceramente, do modo que se finge que se é, para conquistar. Lembre-se que "conquistar", no amor, não é como numa guerra em que se passa a exercer domínio sobre a conquista. Conquistar, no amor, é obter a retribuição amorosa de forma totalmente livre, sem a menor coação. Porque não é possível se amar por coação, mas apenas fingir que se ama. E isso não tem o menor valor.

Hawking: "Philosophy is dead. Scientists have become the bearers of the torch of discovery in our quest for knowledge." O que acha dessa frase absurda?‎

Discordo completamente. Há e não há como não haver, muitas questões que escapam ao domínio da Ciência, isto é, que sejam "metacientíficas". Isso abrange temas éticos, estéticos, epistemológicos, lógicos, metafísicos e outros que pertencem ao escopo da Filosofia por sua natureza. Claro que, sempre que se puder abordar algum tema pela via científica, isso é preferível, porque a Ciência possui critérios de aferição da verdade mais poderosos do que a Filosofia. De fato, ao longo da História, muitos temas tidos como filosóficos passaram para o domínio científico. Isso é bom. Mas não significa que todo o conhecimento venha a ser científico. Além de que a Filosofia não se ocupa apenas com "conhecimentos".

"É melhor correr o risco de salvar um homem culpado do que condenar um inocente". Concorda?‎

Certamente que sim. Ou seja, antes absolver um culpado do que condenar um inocente. Esse é o princípio básico da aplicação de penas pela justiça. Por isso sou totalmente contra a pena de morte.

Por que eu me apaixono muito facilmente ? Na sua concepção, isso seria algo "ruim"?‎

Não sei a razão, mas penso que possa ser o fato de você ser uma pessoa muito aberta a acolher o amor e a paixão, de modo totalmente emocional e sem ponderar razões de senso comum que poderiam levar à rejeição de alguma paixão por motivos alheios à própria paixão. Não acho que isso seja, em absoluto, nada ruim. Pelo contrário, acho que seja, justamente, muito bom. Quanto mais a pessoa for capaz de amar e de se apaixonar, melhor. Mesmo que seja por mais de uma outra ao mesmo tempo. Não tem problema nenhum. O que não se pode é enganar às pessoas dizendo que se ama ou está apaixonado só por uma delas quando não se está. Se todos estiverem felizes com isso, é ótimo. Se não concordarem com uma relação plurívoca, então que não se relacionem. Isso, contudo, não impede que se continue apaixonado, mesmo que sem retribuição. É melhor amar sem ser correspondido do que não amar.

O que achas da teoria dos buracos brancos?‎

Trata-se de uma hipótese advinda da extensão da solução do Buraco Negro de Kerr para regiões implausíveis. Todavia, essa extensão não tem garantia nenhuma de corresponder a algo real, a não ser que alguma observação demonstre que isso exista. Tal comprovação não foi obtida, de modo que sua existência é apenas conjectural. O mesmo acontece com universos paralelos. Veja isto:
https://pt.scribd.com/doc/221824251/Buracos-Negros

Quando um buraco negro suga algo como uma estrela por exemplo, para onde ela vai? Sai por algum outro lugar tipo um buraco branco e para outro universo?‎

Ela fica no caroço do Buraco Negro, de que passa a fazer parte. Não vai para outro lugar do Universo e nem para outro Universo, o que não existe. Como não existem buracos brancos.

Você é fisicalista a priori ou a posteriori?‎

A posteriori. Ou seja, cheguei à conclusão que a realidade objetiva é apenas física a partir de estudos e reflexões sobre as doutrinas religiosas, filosofia, história, cosmologia, evolução, neurociências. Isso se deu entre meus 19 e 24 anos. Antes eu fora um católico fiel e pio. Mas tornei-me, a princípio deísta, depois agnóstico, finalmente ateu da modalidade cética, nem gnóstico nem agnóstico. Fisicalista e reducionista. Além de anarco-comunista. Mas não socialista de estado e nem revolucionário. Minha tese é do atingimento do anarco-comunismo por uma evolução a ser promovida pela educação.

A ciência não consegue provar o método científico. Por isso precisa da Filosofia. Certo?

Sim, mas não só por isso. Aliás, o dito "método científico", de Poper, não é uma unanimidade nem entre filósofos nem entre cientistas. De qualquer modo a ciência requer alguma meta-ciência para direcionar a forma como deve ser feita, bem como validar o que desenvolve. E essa meta-ciência é a filosofia, no caso ou seu ramo epistemológico. Além disso o fazer científico tem outros aspectos, como o ético, que também é papel da filosofia disponibilizar. A metafísica, por exemplo, também fornece à ciência a categorização da realidade. A filosofia da ciência cuida, entre outros assuntos, da divisão e classificação da ciência, bem como da aceitação de alguma explicação como científica ou não (caso este último que, não necessariamente, invalida uma explicação). Por outro lado a filosofia se socorre da ciência em muitos aspectos. Por exemplo, a derrubada da necessidade metafísica de causa para qualquer evento é decorrente da observação científica de eventos sem causa. Por isso é importante que cientistas tenham bons conhecimentos filosóficos e que filósofos tenham conhecimentos científicos.

Professor, sou um cara bastante inseguro. Ex: se uma mina fala que ta afim de mim, a primeira perg que me vem, é: POR QUE EU?. Tem tantos caras mais bonitos por aí. Você acha que essa insegurança é normal? Seria um complexo de inferioridade? Abç.‎

Não é normal não. Mas é possível ser superada. Para começar convença a si mesmo de que tem qualidades. Isso é fácil. Compare-se com outras pessoas. Você verá que há várias melhores que você, mas há muitas mais ainda, piores do que você. Então desenvolva sua auto-estima. Mas sem gabolice. Isto é, sem se considerar o gostosão do pedaço. Seja, simplesmente, realista. Você nem é o melhor de todos nem é desprezível. É uma pessoa normal. Pessoas normais também são apreciadas, admiradas e amadas. Aliás, assim é que é a maior parte das pessoas.

Você se acharia ridículo caso sentisse falta de uma pessoa que te magoou muito?‎

Não. Isso é perfeitamente normal. Se se sente falta de alguém é porque a presença desse alguém supre carências profundas da pessoa. Mesmo que esse alguém tenha magoado a pessoa, isso não significa que sua presença, por isso, deixe de preencher aquelas carências. O que acontece é que, estando magoada, a pessoa rejeita quem a magoou e, com isso, fica sem quem supra suas carências, até achar outra pessoa ou deixar de rejeitar quem a magoou. O ideal é desenvolver uma característica da personalidade que a deixe imune a mágoas. Isto é, que não se importe em sofrer alguma desconsideração, ser vítima de alguma maldade ou algo do tipo. Claro que isso tem seus limites, mas, muitas vezes, a pessoa é muito melindrosa, ficando magoada por motivos de gravidade não tão grande. Quem se liberta disso, torna-se uma pessoa muito mais feliz. Além de que, muitas vezes, a ocorrência que provocou a mágoa não venha a ser por culpa de quem a provocou.

Qual a diferença do pensar do cientista para o do filósofo?‎

O cientista busca construir modelos explicativos da realidade fática, seja física, química, biológica, psicológica, social, política, econômica ou o que seja. Ele busca esquemas de relações entre as ocorrências, bem como explicações para o modo como se estruturam os sistemas de cada um desses níveis da realidade. Tais modelos explicativos têm que ser capazes de prever o comportamento do sistema em estudo, dadas que sejam as condições e o que force o sistema a sofrer alguma transformação. O modelo, pois, precisa ser testado em confronto com a realidade e só é legítimo se o que ele prevê esteja de acordo com o que, de fato, aconteça. Assim o método científico de formular explicações possui duas fases. A primeira é a de se propor um modelo e a segunda de se testar o modelo. A formulação de hipótese se baseia nos conhecimentos já existentes e na intuição do cientista, advinda de uma série de observações e experimentações realizadas sobre a questão. O teste da hipótese se faz considerando situações em que só a explicação proposta seja capaz de dar conta do que acontece, de modo que sem a interveniência da razão explicadora, o fenômeno não ocorra. Em Filosofia a situação é outra. Não há propriamente um fenômeno ou uma estrutura a ser explicada, na realidade objetiva, isto é, fora das mentes. Os fatos e as coisas de que trata a Filosofia são abstrações. Por exemplo, não se cogita das causas de algum fenômeno, mas do fato de causa ser ou não uma necessidade. Não se cogita de medir o tempo, mas do significado do que seja o tempo. Não se cogita de alterar os cromossomos de um ser para provocar uma evolução artificialmente projetada, mas do que significa que algo seja vivo ou não. Então o tipo de raciocínio que se tem que fazer não é como o da ciência, mas sim uma "especulação", isto é, uma proposta de explicação não advinda de observações e experimentações empíricas, mas de uma articulação racional de pensamentos, isto é, de um raciocínio. De qualquer modo, o que se deseja e que as explicações filosóficas sejam fechadas, isto é, que deem conta de explicar e o façam de modo unívoco, isto é, de modo que outras alternativas de explicação sejam rejeitadas. Nem sempre se consegue tal intento e a busca disso é uma das principais tarefas do filósofo.

Dá pra trabalhar como pesquisador em outro país sendo formado em letras-inglês?‎

Sim, você pode ser um professor de português para falantes de inglês. Como em qualquer caso, isso vai depender do fato de você ser, realmente, bom de serviço, bem como de outras coincidências e oportunidades. Ou pode ser um tradutor em tempo real, para conferências. Ou um tradutor de agências de notícias. Ou um tradutor de editoras de livros. Ou um tradutor de programas de computador ou de jogos. Há, de fato, muitas oportunidades, quando se é um ótimo profissional.

O tempo é uma ilusão? Passado, presente e futuro não existem?‎

Não é não. Tempo é algo real. Passado, presente e futuro existem. Tempo é um fluxo de mudanças no estado do Universo. Se nada mudar, o tempo não passa. A sucessão de situações diferentes configura-se em algo que está acontecendo. Daí emerge o tempo, como sendo o elemento que acolhe tais ocorrências. O tempo, mesmo sendo real, não é substancial e nem geométrico. É outra categoria de realidade. Note que não se pode confundir o tempo físico com a percepção psicológica do tempo. O tempo físico decorre independentemente de ser percebido. Veja isto:
http://www.ruckert.pro.br/blog/?page_id=1866

Ernesto, dá para se ser muito bom na formação formal sem abdicar de leituras extracurriculares? Fico com a sensação de que, em meu tempo livre, devo estudar ao máximo os assuntos desenvolvidos em sala, e acabo empacando na leitura de livros não didáticos quando o período letivo começa.‎

Claro que dá. Em minha vida escolar toda sempre me dediquei a estudar vários assuntos extra-curriculares. O segredo é aprender tudo que seja curricular dentro da própria aula, o que dispensa muito estudo fora da aula. Para tal é preciso muita concentração e nenhuma vergonha de pagar mico por perguntar o que não entendeu até entender. Fazendo assim é possível levar um curso qualquer estudando apenas de um terço a meia hora por dia para cada aula que se teve no dia, fora da aula. Então se fica com um vasto tempo para lazer, que pode ser usado para estudar outros assuntos. Veja o meu esquema, que sempre passo para meus estudantes:
Horas da semana: 168
Horas de sono: 52
Horas de aulas: 32
Horas de estudo: 16
Manutenção de vida: 18 (comer, banhar, vestir etc)
Locomoção: 12
Lazer: 38
É só não ficar muito à toa. Mas ainda sobra tempo para namorar. É só ver pouco televisão.

Professor, o quão perto estamos de um reator de fusão nuclear para substituir as outras formas de geração de energia? Na próxima década, é possível que já tenha alguns em pleno funcionamento?‎

Acho que uma década é pouco. Vejo isso para meio ou três quartos de século. O ideal é que se desenvolvam reatores pequenos para geração doméstica de energia, dispensando usinas e redes de distribuição. Como se fosse uma bateria, do tamanho de um botijão de gás. Ou maior, se fosse para um prédio.

Como se estuda sem meramente decorar o livro? Seguindo o seu conselho de preparar uma apostila para estudar anatomia, por exemplo, sinto-me extremamente idiota, como se estivesse apenas transcrevendo o livro para uma folha diferente, com o texto levemente alterado.‎

Mas não é assim. Você tem que absorver o conhecimento de múltiplas fontes e, então, escrever uma apostila de sua própria lavra, inventando o seu próprio modo de exposição e argumentação. Isso te obriga a saber a fundo o que está escrevendo. Tenho feito isso ao longo de toda a minha vida como professor e sempre tive excelentes resultados, pois passava a dominar o assunto com total confiança. O único inconveniente é que dá muito mais trabalho.

O que te atrai?‎

A bondade, a beleza e o conhecimento. Nessa ordem. Riqueza, fama e poder não me atraem.

Dos países de língua inglesa, qual o melhor para um Brasileiro imigrar e ir trabalhar como pesquisador?‎

Meu filho é pesquisador da Minnesota State University. Todavia, acho que a Inglaterra ou a Escócia sejam melhores lugares para se trabalhar como pesquisador. A razão é que a cultura, na Europa, é mais sofisticada. Além de se poder conhecer os demais países com muito mais facilidade. Mesmo a filosofia de trabalho européia é mais proveitosa. Em minha opinião é a melhor forma de se trabalhar no exterior. E se consegue o visto de permanência, como meu filho. Isso não deixa de ser imigração, só que para um trabalho muito mais qualificado.

O que acha? Acredita em Universo Paralelo? https://www.youtube.com/watch?v=bXWS3Tc9q1w‎

Não é uma questão de acreditar e sim de saber se existe ou não. Há dois tipos de Universos Paralelos: o quântico e o relativístico. Tal tipo de coisa é uma suposição com base em extensão das soluções das equações. Todavia não há confirmação fática de que exista outro universo além deste.

"A razão é que o treinamento científico faz o cientista sempre duvidar e não aceitar o senso comum como válido sem um exame minucioso dessa validade. " Concordo, mas vale ressaltar que existe MUITOS, mas MUITOS cientistas que apesar de fazerem ciência "mecanicamente", não se preocupam (continua)‎ JJ (continuação) em desenvolver essa habilidade. É por isso que vemos cientistas que ainda assim acreditam muito em religião e pseudo-ciência. O pensamento científico não é de posse só dos cientistas, e muitos dos próprios cientistas não o utilizam, infelizmente

Isso é um fato realmente deplorável. Os próprios cursos de mestrado e doutorado nem sempre desenvolvem as qualidades desejadas para um cientista, o que deveria ser a norma. Inclusive na disciplina "Metodologia da Pesquisa", em geral só se aborda normas para redação de trabalhos, de citação bibliográfica e outros detalhas formais que não se constituem na essência do que seja pesquisar. Até mesmo o ofício de cientista que, principalmente, consiste em formular hipóteses explicativas de fenômenos, é relegado a um segundo plano, focando-se nos métodos para se testar as hipóteses. Ora, isso é importante, sem dúvida, mas em segundo lugar. Em primeiro lugar se coloca a formulação das hipóteses a serem testadas. Isso é que requer inteligência, engenho, criatividade, descortino, domínio de vastos conhecimentos da área, sagacidade, e tudo o mais que faz diferença para se ser cientista. Nada disso é desenvolvido na pós-graduação. Teria que fazer parte dessa disciplina e teria que ser algo que se visse ao longo do curso todo. Esses detalhes de citações e formatação de originais não precisam de curso nenhum para se saber. É só consultar as normas. Claro que há pessoas que não são cientistas que têm uma forma científica de pensar e cientistas que não têm. Mas estatisticamente, a fração de pessoas que pensam cientificamente entre os cientistas é maior do que entre os não cientistas.

LinkWithin

Related Posts with Thumbnails